SlideShare a Scribd company logo
1 of 12
Download to read offline
www.VNMATH.com

                                            CHUYÊN ð DÃY S                  (BDHSG)

1. KHÁI NI M DÃY S
1) Cho A là m t t p con khác r ng c a t p s nguyên ℤ, hàm s                   u:A → ℝ
                                                                                n ֏ u(n) = u n
ñư c g i là m t dãy s , và kí hi u là (u n ) ho c {u n } . Thông thư ng ta hay ch n A sao cho ph n t nh nh t
c a A là 1. Dãy (un) g i là dãy s h u h n (ho c dãy s vô h n) n u A là t p h p g m h u h n (vô h n) ph n t .
S un ñư c g i là s h ng t ng quát c a dãy (un).
2) Dãy s (un) ñư c g i là dãy s tăng (tăng không nghi m ng t, gi m, gi m không nghiêm ng t) n u u n < u n +1
(tương ng u n ≤ u n +1 , u n > u n +1 , u n ≥ u n +1 ) v i m i n ∈ A.
3) Dãy s (un) ñư c g i là tu n hoàn n u t n t i s nguyên dương k sao cho u n + k = u n , ∀n ∈ A. S k nh nh t
tho mãn tính ch t này ñư c g i là chu kì c a dãy tu n hoàn (un). N u k = 1 thì ta ñư c m t dãy h ng (t t c các
s h ng b ng nhau).
4) Dãy s (un) ñư c g i là b ch n trên n u t n t i s th c M sao cho u n ≤ M v i m i n ∈ A. Dãy s (un) ñư c
g i là b ch n dư i n u t n t i s th c m sao cho u n ≥ m v i m i n ∈ A. Dãy s (un) ñư c g i là b ch n (ho c
gi i n i) n u nó v a b ch n trên v a b ch n dư i, t c là t n t i s th c M, m sao cho m ≤ u n ≤ M v i m i
n ∈ A, ho c t n t i s th c C sao cho u n ≤ C, ∀n ∈ A. Dãy s h u h n ho c tu n hoàn thì luôn b ch n.
2. C P S
1) C p s c ng
- Dãy s (un) ñư c g i là c p s c ng n u m i s h ng ñ u tho mãn u n +1 − u n = d (d: h ng s , g i là công sai).
-  Công th c truy h i: u n +1 = u n + d. Công th c s h ng t ng quát: u n = u1 + (n − 1)d, ∀n ∈ A. Công th c tính
                                       n           n                       n(n − 1)
   t ng n s h ng ñ u tiên: Sn = (u1 + u n ) = (2u1 + (n − 1)d) = nu1 +              d. Tính ch t các s h ng:
                                       2           2                          2
    u k +1 + u k −1 = 2u k .
2) C p s nhân
- Dãy s (un) ñư c g i là c p s nhân n u m i s h ng ñ u tho mãn u n +1 = u n .q (q: h ng s , g i là công b i).
-   Công th c truy h i: u n +1 = u n .q. Công th c s h ng t ng quát: u n = u1.q n −1. Công th c tính t ng n s h ng
                                              1 − q n +1                                                    2
    ñ u tiên: Sn = nu1 n u q = 1, Sn = u1                n u q ≠ 1. Tính ch t các s h ng: u k +1.u k −1 = u k .
                                                1− q
3) C p s nhân c ng
- Dãy s (un) ñư c g i là c p s nhân c ng n u m i s h ng ñ u tho mãn u n +1 = u n .q + d (q, d là h ng s ).
4) C p s ñi u hoà
                                                                                                 2u n −1u n +1
- Dãy s (un) ñư c g i là c p s ñi u hoà n u m i s h ng c a nó ñ u khác 0 và tho mãn u n =                       ,
                                                                                                u n −1 + u n +1
        1 1 1           1
   hay    = (      +       ). (H c sinh t ôn t p các d ng toán v c p s )
       u n 2 u n −1 u n +1

3. XÁC ð NH S H NG T NG QUÁT C A DÃY S
3.1. D ðOÁN S H NG T NG QUÁT VÀ CH NG MINH B NG QUI N P
BÀI T P
1) Xác ñ nh s h ng t ng quát c a dãy s cho b i:
                   u
a)u1 = 1, u n +1 = n , ∀n = 1, 2,3,...          b)u1 = 2, u n +1 = 2 + u n , ∀n = 1, 2,3,...
                  1+ un
                    3 2 5                                              3 −1            3 − 1 + ( 3 + 1)u n
c)u1 = 1, u n +1 = − u n + u n + 1, ∀n = 1, 2,3,...           d)u1 =        , u n +1 =                     , ∀n = 1, 2,3,...
                    2     2                                            3 +1            3 + 1 − ( 3 − 1)u n
                                                                                                               xa.nguyenvan@gmail.com
www.VNMATH.com
TÂM SÁNG – CHÍ B N

       1                                                          1
e)u1 = , u n +1 = 2u n − 1, ∀n = 1, 2,3,...
                        2
                                                          f )u1 = , u n +1 = 2u n 1 − u n , ∀n = 1, 2,3,...
                                                                                            2
       2                                                          2
3.2. M T S D NG TRUY H I ð C BI T
   V i dãy s cho b i công th c truy h i d ng u n +1 = u n + f (n) thì u n = u1 + f (1) + f (2) + ... + f (n − 1).
   V i dãy s cho b i công th c truy h i d ng u n +1 = u n .g(n) thì u n = u1.g(1).g(2)...g(n − 1).
BÀI T P
2) Xác ñ nh s h ng t ng quát c a dãy s cho b i:
                                                                           (n + 1) 2 u n
a)u1 = 1, u n +1 = u n + n!.n, ∀n = 1, 2,3,...          b)u1 = 1, u n +1 =               , ∀n = 1, 2,3,...
                                                                            n(n + 2)
c)u1 = 1, u n +1 = u n + n 3 + 3n 2 − 3n + 1, ∀n = 1, 2,3,...   d)u1 = 3, u n +1 = u n + 3n , ∀n = 1, 2,3,...
                                                                                          1
e)u1 = 1, u n +1 = u n + (n + 1).2n , ∀n = 1, 2,3,...           f )u1 = 2, u n +1 = 2 −      , ∀n = 1, 2,3,...
                                                                                          un

                 n2 −1                                                                n
g)u1 = 1, u n +1 =     u n , ∀n = 1, 2,3,...                    h)u1 = 0, u n +1 =        (1 + u n ), ∀n = 1, 2,3,...
                  n                                                                  n +1
3.3. PHƯƠNG TRÌNH ð C TRƯNG
Ta ch xét hai trư ng h p ñơn gi n sau ñây:
a) Xét dãy s (un) cho b i u1, u 2 , u n + 2 = a.u n +1 + b.u n , ∀n ∈ ℕ * (a, b = const). Khi ñó phương trình x 2 − ax − b = 0
ñư c g i là phương trình ñ c trưng c a dãy s ñã cho.
   N u phương trình trên có hai nghi m th c phân bi t x1, x 2 thì u n = A.x1 + B.x n . n
                                                                                              2
   N u phương trình trên có hai nghi m th c trùng nhau x1 = x 2 thì u n = (A + nB).x1 .
                                                                                     n

   N u phương trình trên có ∆ < 0 , g i hai nghi m ph c c a nó là x1 , x 2 , và bi u di n hai s ph c này                             d ng
lư ng giác x1 = r(cos ϕ + i.sin ϕ), x 2 = r(cos ϕ − i.sin ϕ), v i r, ϕ là các s th c, r là môñun c a x1 và x 2 ,
ϕ ∈ [ 0; 2π ) , i là ñơn v o, thì u n = r n (A.cos nϕ + B.sinnϕ). ( ñó các h ng s A, B ñư c xác ñ nh nh u1, u 2 )
b) Xét dãy s (un) cho b i u1, u 2 , u 3 , u n +3 = a.u n + 2 + b.u n +1 + c.u n , ∀n ∈ ℕ * (a, b, c = const) có phương trình ñ c
trưng x 3 − ax 2 − bx − c = 0.
   N u phương trình trên có ba nghi m th c phân bi t x1, x 2 , x 3 thì u n = A.x1 + B.x n + C.x 3 .
                                                                                n
                                                                                        2
                                                                                                n

   N u phương trình trên có ba nghi m th c x1 , x 2 , x 3 mà x1 ≠ x 2 = x 3 thì u n = A.x1 + (B + nC).x 2 .
                                                                                         n              n

   N u phương trình trên có ba nghi m th c x1 , x 2 , x 3 và x1 = x 2 = x 3 thì u n = (A + nB + n 2C).x1 .
                                                                                                       n

    N u phương trình trên có ba nghi m x1, x 2 , x 3 trong ñó x1 là nghi m th c, còn hai nghi m
x 2 = r(cos ϕ + i.sin ϕ), x 3 = r(cos ϕ − i.sin ϕ) là hai nghi m ph c (không ph i là s                     th c) thì
u n = A.x1 + r n (B.cos nϕ + C.sinnϕ). ( ñó các h ng s A, B, C ñư c xác ñ nh nh u1, u 2 , u 3 )
         n



VD1. Cho dãy s (u n ) xác ñ nh b i u1 = 1, u 2 = 0, u n + 2 = u n +1 − u n , ∀n ∈ ℕ *. Ch ng minh (u n ) b ch n.
                                                                                                             π   π
HD. Phương trình ñ c trưng c a dãy s ñã cho là x 2 − x + 1 = 0 có hai nghi m ph c x1 = cos + i.sin ,
                                                                                                             3   3
         π        π                     nπ          nπ
x 2 = cos − i.sin , nên u n = 1n (A.cos    + B.sin ), ∀n ∈ ℕ *. Do u1 = 1, u 2 = 0, nên ta có
         3        3                      3           3
          π         π                   A B 3
1 = A.cos 3 + B.sin 3      (= u1 )       +          =1          A = 1
                                        2      2                
                                    ⇔                      ⇔             3.
0 = A.cos 2π + B.sin 2π    (= u 2 )     − A + B 3 = 0           B =

                                         2                              3
            3          3                          2
                                                                                                                        xa.nguyenvan@gmail.com
www.VNMATH.com
TÂM SÁNG – CHÍ B N


                     nπ    3     nπ                         nπ    3      nπ          3     2
Suy ra u n = cos        +    .sin , ∀n ∈ ℕ *. V y u n = cos    +    .sin    ≤ 12 + ( ) 2 =    , ∀n ∈ ℕ*, hay
                      3   3       3                          3   3        3         3       3
(u n ) là dãy b ch n.
VD2. Cho (u n ) có u1 = 0, u 2 = 16, u3 = 47, u n +3 = 7u n +2 − 11u n +1 + 5u n , ∀n ∈ ℕ *. Tìm dư khi chia u 2011 cho 2011.
HD. Phương trình ñ c trưng x 3 − 7x 2 + 11x − 5 = 0 có 3 nghi m th c x1 = 5 (nghi m ñơn), x 2 = x 3 = 1 (nghi m
                                                                                      1
kép) do ñó u n = A.5n + (B + nC).1n , ∀n ∈ ℕ *. Vì u1 = 0, u 2 = 16, u 3 = 47 nên A = , B = −13, C = 12. Suy ra
                                                                                      5
       n −1
u n = 5 + 12n − 13, ∀n ∈ ℕ *. T ñó u 2011 = 5  2010
                                                    + 12.2011 − 13. Theo ñ nh lí Phécma thì 52010 − 1⋮ 2011 (ñ nh
lí Phécma: N u p là s nguyên t , a là s nguyên và (a, p) = 1, thì a p−1 ≡ 1 (mod p)). V y u 2011 chia cho 2011 dư
 −12 (hay dư 1999).
VD3. Cho hai dãy s (x n ), (y n ) tho mãn x1 = y1 = 1, x n +1 = 4x n − 2y n , y n +1 = x n + y n , ∀n ∈ ℕ *. Xác ñ nh công
th c c a x n , y n .
HD.     Ta    có     x n + 2 = 4x n +1 − 2y n +1 = 4x n +1 − 2(x n + y n ) = 4x n +1 − 2x n − 2y n = 4x n +1 − 2x n + x n +1 − 4x n     hay
x n + 2 = 5x n +1 − 6x n (∀n ∈ ℕ*). Dãy (u n ) có phương trình ñ c trưng x 2 − 5x + 6 = 0 ⇔ x = 2 ho c x = 3. Suy ra
                                                                        1
x n = A.2n + B.3n , ∀n ∈ ℕ *. Mà x1 = 1, x 2 = 4x1 − 2y1 = 2, nên A = , B = 0, và ta có x n = 2n −1, ∀n ∈ ℕ *. T
                                                                        2
                                     n −1                n −1
ñó và x n +1 = 4x n − 2y n ⇒ y n = 2 . V y x n = y n = 2 , ∀n ∈ ℕ *.

3.4. PHƯƠNG PHÁP DÃY S PH
VD4. Cho dãy s (u n ) : u1 = 1, u 2 = 2, u n + 2 = 2.u n +1 − u n + 1, ∀n ∈ ℕ *. ð t v n = u n +1 − u n . Ch ng minh (v n ) là
c p s c ng và tìm u n .
HD. a) Ta có            v1 = u 2 − u1 = 1. Và u n +2 = 2.u n +1 − u n + 1, ∀n ∈ ℕ* ⇔ u n + 2 − u n +1 = u n +1 − u n + 1, ∀n ∈ ℕ *
⇔ v n +1 = v n + 1, ∀n ∈ ℕ *. V y (v n ) là c p s c ng v i s h ng ñ u tiên v1 = 1, công sai d = 1.
b) T câu a ta có vn = v1 + (n −1).d = 1+ (n −1).1 = n, hay un+1 − un = n, ∀n ∈ ℕ *. Suy ra un = (un − un−1) + (un−1 − un−2 ) +
                                                                        (n − 1).n n 2 n
+... + (u 2 − u1 ) + u1 = [(n − 1) + (n − 2) + ... + 2 + 1] + 1 = 1 +            =   − + 1, ∀n ∈ ℕ *.
                                                                            2      2 2
                                                            2          1
VD5. Cho dãy s           (u n ) : u1 = 0, u 2 = 1, u n + 2 = .u n +1 + u n , ∀n ∈ ℕ *. ð t v n = u n +1 − u n . Ch ng minh (v n ) là
                                                            3          3
c p s nhân và tìm u n .
                                                                       2          1
HD.       Ta        có      v1 = u 2 − u1 = 1.        Và      u n + 2 = .u n +1 + u n , ∀n ∈ ℕ* ⇔ 3u n + 2 = 2u n +1 + u n , ∀n ∈ ℕ* ⇔
                                                                       3          3
                                                                −1
3(u n + 2 − u n +1 ) = −(u n +1 − u n ), ∀n ∈ ℕ * ⇔ v n +1 = v n , ∀n ∈ ℕ *. V y (v n ) là c p s nhân v i s h ng ñ u tiên
                                                                 3
                               1
v1 = 1, công b i q = − .
                               3
                            1
Ta có vn = v1.qn−1 = (− )n−1, ∀n ∈ℕ *. Suy ra un = (un − un−1) + (un−1 − un−2) ++... + (u2 − u1) + u1 = vn−1 + vn−2 +
                            3
                                                                                 1
                                                                         1 − (− ) n −1
                                      1       1               1                          3      9
+... + v 2 + v1 + u1 = 0 + 1 + (− ) + (− ) 2 + ... + (− ) n − 2 = 1.             3      = +         , ∀n ∈ ℕ *.
                                      3       3               3            1 − (− )
                                                                                   1     4 4.(−3) n
                                                                                   3
VD6. Tìm s h ng t ng quát c a dãy s ( un ) có u1 = c, un+1 = q.un + an + d , ∀n ∈ ℕ*, ñó a, c, d, q là h ng s .

                                                                                                                        xa.nguyenvan@gmail.com
www.VNMATH.com
TÂM SÁNG – CHÍ B N


HD. V i q = 1 thì un+1 = un + an + d , ∀n ∈ ℕ * . Ta nh n th y u 2 = u1 + a + d, u 3 = u 2 + 2a + d,...,
un−1 = un −2 + (n − 2)a + d,un = un−1 + (n −1)a + d ⇒ u2 + u3 + ... + un−1 + un = u1 + u2 + ... + un−2 + un−1 +
                                                           n(n −1)                        n(n −1)
+a(1 + 2 + ... + (n − 2) + (n −1)) + (n −1)d ⇒ un = u1 +           a + (n −1)d ⇒ un = c +         a + (n −1)d, (n ∈ ℕ*).
                                                              2                              2
Khi q ≠ 1, ta s xét m t dãy ph ( vn ) th a mãn un = vn + bn + e,              n ∈ ℕ*, ñó b, e là nh ng h ng s , và ta c
g ng ch n b, e thích h p ñ ( vn ) là c p s nhân. T                               ñ ng th c truy h i ban ñ u ta có
vn +1 = qvn + (qb + a − b)n + (qe + d − b − e), n ∈ ℕ*, và d th y ñ ( vn ) là c p s nhân thì c n có
qb + a − b = qe + d − b − e = 0 ⇔ b = a , e = d − a − qd ,(q ≠ 1) . Lúc này (v i b, e như trên) do vn +1 = qvn
                                               1− q   2
                                                            (q − 1)
nên ( vn ) là c p s nhân có công b i q. Suy ra vn =         v1 .q n−1 = (u1 + aq + dq − d ).q n−1 , t ñó ta tính ñư c s h ng
                                                                               (q − 1)2

                                     aq + dq − d n −1 a.n d −a−qd
t ng quát c a ( un ) là un = (u1 +              ).q +    +        (n ≥ 2).
                                       (q − 1)2       1−q (q −1)2
                                                            n(n − 1)
                                                           c + 2 a + (n − 1)d ,
                                                           
                                                                                                  khi q = 1
V y, s h ng t ng quát c a ( un ) ñã cho là :          un =                                                 .
                                                            (c +
                                                                  aq + dq − d n−1 a.n d −a−qd
                                                                             ).q +    +        , khi q ≠ 1
                                                           
                                                                   (q − 1)2       1−q (q −1)2
BÀI T P
                                3 + 2u n                        u −1
3) Cho (u n ) : u1 = 0, u n +1 =         , ∀n ∈ ℕ *. ð t v n = n       . Ch ng minh (v n ) là c p s nhân và tìm u n .
                                 4 + un                         un + 3
                    1           (n + 1)u n                        u
4) Cho (u n ) : u1 = , u n +1 =             , ∀n ∈ ℕ *. ð t v n = n . Ch ng minh (v n ) là c p s nhân và tìm u n .
                    3               3n                             n
5) Cho (u n ) : u1 = 1, u n +1 = u n + 2.(1 + 3 ), ∀n ∈ ℕ *. ð t v n = u n − 3n. Ch ng minh dãy s (v n ) là c p s
                                                n

    c ng và tìm u n .
6) Nêu cách xác ñ nh s h ng t ng quát c a dãy s ( u n) cho b i u1 = c, un+1 = q.un + P (n), ∀n ∈ ℕ * ,                     ñó c, q
   là các h ng s , còn P(n) là m t ña th c b c k cho trư c.

3.5. TUY N TÍNH HOÁ M T S                DÃY PHI TUY N
                                                px + q
a) V i dãy s (x n ) cho b i x1 = a, x n +1 = n             , ∀n ∈ ℕ*, và a, p, q, r, s là các h ng s , ta xét hai dãy (u n ),
                                                rx n + s
                                                                                               u
(v n ) tho mãn u1 = a, v1 = 1, u n +1 = pu n + qv n , v n +1 = ru n + sv n , ∀n ∈ ℕ*, thì x n = n . T ñó tìm ra u n , v n , x n .
                                                                                               vn
                                                  x2 + d
b) V i dãy s (x n ) cho b i x1 = a, x n +1 = n             , ∀n ∈ ℕ*, và a, d là các h ng s , d ≠ 0, ta xét hai dãy (u n ),
                                                    2x n
                                                                                           u
(v n ) tho mãn u1 = a, v1 = 1, u n +1 = u n + dv 2 , v n +1 = 2u n v n , ∀n ∈ ℕ*, thì x n = n . T ñó tìm ra u n , v n , x n .
                                          2
                                                 n
                                                                                           vn
                                            xn                                                  1
VD7. Cho dãy (x n ) có x1 = 1, x n +1 =            , ∀n ∈ ℕ*, tìm x n và ch ng minh x n < , ∀n ≥ 2.
                                          2 + xn                                                n
                                                                                                               u
HD. Xét hai dãy (u n ), (vn ) tho mãn u1 = v1 = 1, u n +1 = u n , v n +1 = u n + 2v n , ∀n ∈ ℕ*, thì x n = n . Ta th y
                                                                                                               vn
x n = 1, ∀n ∈ ℕ *. Và vn+1 = 2vn +1, ∀n ∈ ℕ* ⇔ vn+1 +1 = 2(vn +1), ∀n ∈ ℕ*. Suy ra vn +1 = 2.(vn −1 +1) = 22 (vn −2 +1) =
                                                                                                               xa.nguyenvan@gmail.com
www.VNMATH.com
TÂM SÁNG – CHÍ B N


                                                               u     1
= ... = 2n −1 (v1 + 1) = 2n ⇒ v n = 2n − 1, ∀n ∈ ℕ *. V y x n = n =     , ∀n ∈ ℕ *. Ta có 2n = C0 + C1 + ...Cn >
                                                                                                n    n
                                                                                                             n
                                                                    n
                                                               vn 2 − 1
                                              1     1
> C0 + C1 = 1 + n, ∀n ≥ 2, t c là x n =
     n     n                                      < , ∀n ≥ 2.
                                             n
                                            2 −1 n
                                              2x n + 1
VD8. Cho dãy (x n ) có x 0 = 2, x n +1 =               , ∀n ∈ ℕ, tìm x n và tìm ph n nguyên c a S = x1 + x 2 + ... + x n .
                                              2 + xn
                                                                                                                              u
HD. Xét hai dãy (u n ), (v n ) tho mãn u 0 = 2, v0 = 1, u n +1 = 2u n + v n , v n +1 = u n + 2v n , ∀n ∈ ℕ, thì x n = n . Ta
                                                                                                                              vn
có u1 = 2u 0 + v0 = 5, u n + 2 = 2u n +1 + vn +1 = 2u n +1 + u n + 2vn = 2u n +1 + u n + 2(u n +1 − 2u n ) ⇒ u n + 2 = 4u n +1 − 3u n ,
                                                                                                                           1       3
phương trình x 2 − 4x + 3 = 0 ⇔ x = 1, x = 3, nên u n = A + B.3n , ∀n ∈ ℕ. Do u 0 = 2, u1 = 5 nên A = , B = .
                                                                                                                           2       2
                   3n +1 + 1                                        3n +1 − 1                                 3n +1 + 1
Suy     ra    un =           , ∀n ∈ ℕ.      Tính     ñư c      vn =           , ∀n ∈ ℕ.       V y      xn =       , ∀n ∈ ℕ. ð t
                       2                                                2                               3n +1 − 1
                           32 + 1 33 + 1         3n + 1 3n +1 + 1                 3n +1 + 1             2
S = x1 + x 2 + ... + x n =       +       + ... +        +          . Lưu ý x n =             = 1+            > 1, ∀n ∈ ℕ. Ta có
                            2        3            n        n −1                     n +1             n +1
                           3 −1 3 −1             3 −1 3         −1                3      −1        3      −1
                                                                                                 1
(1 + 2)n +1 = C0 +1 + C1 +1.2 + Cn +1.22 + ... + Cn +1.2n +1 ≥ C0 +1 + C n +1.22 = 1 + (n + 1)n. .4 = 2n(n + 1) + 1, ∀n ∈ ℕ *.
                n        n
                                   2               n +1
                                                                 n
                                                                         2
                                                                                                 2
                                                              2              1           1     1
D n t i 3n +1 − 1 ≥ 2n(n + 1), ∀n ∈ ℕ* ⇒ x n = 1 +                  ≤ 1+          = 1+ −           , ∀n ∈ ℕ *. Do v y v i
                                                          3n +1 − 1      n(n + 1)        n n +1
                                                 1 1          1 1              1      1                 1
∀n ∈ ℕ * thì S = x1 + x 2 + ... + x n ≤ (1 + − ) + (1 + − ) + ... + (1 + −                ) = n +1−         < n + 1. M t khác
                                                 1 2          2 3              n n +1                 n +1
S = x1 + x 2 + ... + x n > n, ∀n ∈ ℕ *. Như v y n < S < n + 1, ∀ ∈ ℕ*, nên [S] = n, ∀∈ ℕ *.

4. M T S BÀI TOÁN LIÊN QUAN T I TÍNH CH T C A DÃY S
VD9. Cho các s dương a1, a 2 ,..., a13 tho mãn a1 + a 2 + ... + a13 ≥ 13. Ch ng minh dãy (un) cho b i
u n = a1 + a n + ... + a13 , ∀n ∈ ℕ*, là dãy tăng không nghiêm ng t.
       n
             2
                        n

HD. V i m i s dương a và s nguyên dương n ta có (a n − 1)(a − 1) ≥ 0 nên a n +1 − a n ≥ a − 1. T ñó suy ra
u n +1 − u n = (a1 +1 + a n +1 + ... + a13 1) − (a1 + a n + ... + a13 ) ≥ a1 + a 2 + ... + a13 − 13 ≥ 0, ∀n ∈ℕ*, hay un+1 ≥ un , ∀n ∈ ℕ *.
                 n
                          2
                                        n+        n
                                                        2
                                                                   n

V y (un) là dãy s tăng không nghiêm ng t.
                                                                         −1
VD10. Ch ng minh dãy s (un) cho b i u1 = 1, u n =                               , ∀n = 2,3, 4... là dãy s gi m và b ch n.
                                                                     3 + u n −1
                                      −3 + 5                                               −3 + 5
HD. * Trư c h t ta ch ng minh u n >          , ∀n ∈ ℕ *. Th t v y, v i n = 1 thì u1 = 1 >         . Gi s
                                         2                                                    2
     −3 + 5                    3+ 5      1       2      3− 5               −1     −3 + 5
uk >        . Khi ñó 3 + u k >      ⇒       <         =       ⇒ u k +1 =        >        . Theo nguyên lí
        2                        2    3 + uk 3 + 5        2              3 + uk      2
                           −3 + 5                                              −3 + 5
qui n p toán h c, ta có u n >         , ∀n ∈ ℕ*, t c là (un) b ch n dư i b i            .
                                2                                                 2
                                             1     u 2 + 3u n + 1                         −3 + 5
* Bây gi ta xét hi u u n − u n +1 = u n +        = n              > 0, ∀n ∈ ℕ*, do u n >         , ∀n ∈ ℕ *. V y (un)
                                          3 + un       3 + un                                2
là dãy s gi m.
* Vì (un) gi m nên 1 = u1 > u 2 > ... > u n > u n +1 > ... suy ra (un) b ch n trên b i 1. V y (un) là dãy s b ch n.

                                                                                                                       xa.nguyenvan@gmail.com
www.VNMATH.com
TÂM SÁNG – CHÍ B N


                                                                                       −a
Nh n xét. Ta có k t lu n tương t ñ i v i dãy s (un) cho b i u1 = α , u n =                     , ∀n = 2,3, 4... , v i a, b, c
                                                                                   b + cu n −1

                          −b + b 2 − 4ac
dương, b 2 − 4ac > 0, α >                .
                               2c
                                                                          1        a
VD11. Xét tính ñơn ñi u và b ch n c a dãy s (un) có x1 > 0, x n +1 =        (x n + ), ∀n ∈ ℕ*,       ñó a > 0 là h ng s .
                                                                          2       xn
                                   1        a
HD. Do x1 > 0, a > 0 và x n +1 = (x n + ), ∀n ∈ ℕ*, nên b ng qui n p ta ch ng minh ñư c x n > 0, ∀n ∈ ℕ*,
                                   2       xn
                                                                    1           a                         x
t c là dãy (un) b ch n dư i. Áp d ng b t ñ ng th c Côsi ta có x n = (x n −1 +        ) ≥ a , ∀n ≥ 2. Do ñó n +1 =
                                                                    2         x n −1                       xn
  1     a     1 a
 = +        ≤ +      = 1, ∀n ≥ 2. Suy ra (un) là dãy gi m không nghi m ng t, k t s h ng th 2 tr ñi. D th y
  2 2x 2 2 2a
          n
(un) b ch n trên. V y (un) là dãy b ch n.
BÀI T P.
7) Cho dãy (x n ) : x1 = 7, x 2 = 50, x n + 2 = 4x n +1 + 5x n − 1975, ∀n ∈ ℕ *. Ch ng minh x1996 ⋮1997.
8) Cho dãy các s nguyên (x n ) : x1 = 15, x 2 = 35, x 3 = 405, x n + 3 = 6x n + 2 + 13x n +1 − 42x n , ∀n ∈ ℕ *. Tìm nh ng
s h ng c a dãy mà ch s t n cùng c a s h ng ñó là s 0.

5. GI I H N C A DÃY S
    Chúng tôi lưu ý kí hi u n là m t bi n s nguyên dương, còn n0 là m t h ng s nguyên dương (tr trư ng h p
có chú thích c th ).
    M t dãy s có gi i h n h u h n ñư c g i là dãy s h i t , n u nó có gi i h n vô c c ho c không có gi i h n
thì ta nói nó phân kì.
    Khi xét gi i h n c a dãy s (un) ta có th ch xét các s h ng c a dãy k t s h ng th n0 tr ñi, t c là vi c
thay ñ i h u h n s h ng ñ u tiên c a dãy s không làm nh hư ng ñ n tính h i t , và không làm nh hư ng ñ n
gi i h n (n u có) c a dãy s ñó.
    Gi i h n c a dãy s (n u có) là duy nh t. T c là n u limun = u thì limun+k = limun-k = u, v i k là s nguyên
dương tùy ý.
   limun = 0 ⇔ lim|un| = 0.
   limun = u ⇔ lim|un – u| = 0.
    limun = u ⇔ limu2n = limu2n+1 = u .
                                                                           1  1
   N u limun = u thì lim|un| = |u| và lim u k = uk (k nguyên dương), lim
                                            n                                = (u ≠ 0) .
                                                                           un u
   N u un < un+1 < M (∀n ≥ n0) thì (un) h i t , limun = u ≤ M, và un < u (∀n ≥ n0).
   N u un ≤ un+1 ≤ M (∀n ≥ n0) thì (un) h i t , limun = u ≤ M, và un ≤ u (∀n ≥ n0).
   N u un ≤ un+1(∀n ≥ n0) và (un) không b ch n trên thì limun = + ∞.
   N u un > un+1 > m (∀n ≥ n0) thì (un) h i t , limun = u ≥ m, và un > u (∀n ≥ n0).
   N u un ≥ un+1 ≥ m (∀n ≥ n0) thì (un) h i t , limun = u ≥ m, và un ≥ u (∀n ≥ n0).
  N u un ≥ un+1(∀n ≥ n0) và (un) không b ch n dư i thì limun = – ∞.
  M t dãy s h i t thì b ch n.
  N u xn ≤ yn ≤ zn (ho c xn < yn < zn ) v i ∀n ≥ n0, ñ ng th i limxn = limzn = a thì limyn = a (nguyên lí gi i h n
k p).
  N u un ≥ 0 (ho c un > 0) v i ∀n ≥ n0, và limun = u thì u ≥ 0 và lim u n = u .
                                                                                                           xa.nguyenvan@gmail.com
www.VNMATH.com

TÂM SÁNG – CHÍ B N


   Gi s un ≤ vn (ho c un < vn) v i ∀n ≥ n0. Khi ñó:
             • N u limun = u, limvn = v thì u ≤ v.
             • N u limun = + ∞ thì limvn = + ∞.
             • N u limvn = – ∞ thì limun = – ∞.
                          n                                                          un
              1                                                     1 
   Ta có lim 1 +  = e. N u limun = + ∞ ho c limun = – ∞ thì lim 1 +                   = e.
              n                                                  un 
   N u (un) b ch n và limvn = 0 thì lim(unvn) = 0.

                                         2a n −1.a n +1
VD12. Cho dãy s (an) th a mãn a n =                      , ∀ n > 1. Tìm lim an.
                                         a n −1 + a n +1
                                                                      2a k .a k + 2                       2a .a
HD. N u t n t i s nguyên dương k sao cho ak = 0 thì a k+1 =                         = 0 , suy ra a k+2 = k+1 k +3 = 0 , và
                                                                      a k + a k +2                        a k+1 + a k +3
                2a k .a k + 2                                                                          1
d n t i a k+1 =               không có nghĩa. Do v y an ≠ 0, ∀ n ∈ N *. Bây gi ta ñ t un =                  (∀ n ∈ N *), thì un
                a k + a k +2                                                                           an
                                                                  1
≠ 0 (∀ n ∈ N *), thay vào ñ ng th c ñ bài ta ñư c un = (un-1+ un+1), ∀n >1, suy ra (un) là c p s c ng có công
                                                                  2
                                                                                                        a1
sai d, và s h ng t ng quát un = u1 + (n – 1)d ≠ 0, v i m i n ∈ N *. Như v y a n =                                 (∀ n ∈ N *).
                                                                                                 1 + (n − 1)da1
N u d = 0 (⇔ u1 = u2 = …⇔ a1 = a2 = …) thì an = a1(∀ n ∈ N *) và lim an = a1. N u d ≠ 0 (⇔ các s h ng c a
                                                                                             a1
dãy (un) phân bi t ⇔ các s h ng c a dãy (an) phân bi t) thì lim a n = lim                                = 0.
                                                                                      1 + ( n − 1) d a 1
    V y, n u các s h ng c a (an) b ng nhau thì lim an = a1, n u các s h ng c a dãy (an) phân bi t thì lim an = 0.


                                   0, khi q < 1
                             
                                    1, khi q = 1
                          n
VD13. Ch ng minh r ng limq =      + ∞, khi q > 1        .
                             
                             không tô`n tai, khi q ≤ −1
                                          .
                             
HD. N u q = 1 thì có ngay limqn = 1. Và n u q = 0 thì cũng có ngay limqn = 0.
                              1                  1
    N u 0 <|q| < 1 thì          = 1 + a (a > 0)⇒ n = (1 + a)n = C0 + a.C1 + ... + a n .Cn ≥1 + na > 0, v i ∀ n ∈ N *.
                                                                 n      n
                                                                                        n
                              q                 q
                n           1                                        1
Do ñó 0 < q          ≤          , v i ∀ n ∈ N *. Vì a > 0 nên lim        = 0. Theo nguyên lí gi i h n k p thì lim|q|n = 0,
                         1 + na                                   1 + na
hay limqn = 0.
                              1                                      1
    N u q > 1 thì 0 <             < 1 nên theo ch ng minh trên lim     n
                                                                         = 0. Ta ñi ñ n limqn = + ∞.
                              q                                      q
    N u q = –1 thì limq2n = 1 còn limq2n+1 = –1 nên không t n t i limqn.
    N u q < – 1 thì q2 > 1 nên limq2n = lim(q2)n = + ∞, và limq2n+1 = lim[q.(q2)n] = – ∞, vì th không t n t i limqn.

VD14. Cho dãy s (an) th a mãn an ≤ an+1 – a 2 +1 , ∀n ∈ ℕ * . Tìm gi i h n liman.
                                            n


                                                                                                             xa.nguyenvan@gmail.com
www.VNMATH.com
TÂM SÁNG – CHÍ B N


                                                           1
                                                             , ∀n ∈ℕ * . T c là (an) là dãy b ch n trên và không
HD. T an ≤ an+1 – a 2 +1 , ∀n ∈ ℕ * , suy ra an ≤ an+1 và an ≤
                    n
                                                           4
gi m. Do ñó (an) có gi i h n h u h n liman = a. L y gi i h n h n hai v b t ñ ng th c ñ bài cho, ñư c a ≤ a – a2,
hay a = 0. V y liman = 0.
VD15. Cho dãy s (xn) th a mãn x1 = 3, x n+1 − 3x n +1 =                        2 + x n , ∀ n ∈ N *. Tìm gi i h n lim xn.
                                        3


HD. Ta th y x1 = 3 > 2. Gi                             s    xk > 2, lúc này x 3 − 3x k +1 = 2 + x k >
                                                                              k+1                                      2 + 2 = 2 nên
 x 3 − 3x k +1 − 2 > 0 ⇔ (xk+1 + 1)2(xk+1 – 2) > 0 ⇔ xk+1 > 2. T c là xn > 2, ∀ n ∈ N *. Xét hàm s f(t) = t3 – 3t,
   k+1
có f ’(t) = 3t2 – 3 = 3(t + 1)(t – 1) > 0, ∀ t > 2, suy ra f(t) ñ ng bi n trên kho ng (2; + ∞). Ki m tra th y
 x1 − 3x1 = 18
  3
                         >     x 3 − 3x 2 = 5 ⇒
                                 2
                                                     f(x1) >   f(x2)       ⇒ x1 >          x2.   Gi   s   xk   > xk+1 ⇒ 2 + x k >
> 2 + x k+1 ⇒ x k+1 − 3x k +1 > x 3 − 3x k + 2 ⇒ f(xk+1) > f(x k +2) ⇒ xk+1 > xk+2. Do ñó xn > xn+1 v i ∀ n ∈ N *. Dãy
                         3
                                  k+2

(xn) gi m và b ch n dư i b i 2 nên t n t i gi i h n h u h n lim xn = x ≥ 2. L y gi i h n hai v ñ ng th c ñ bài

ta ñư c x3 – 3x =             2 + x ⇔ x6 – 6x4 + 9x2 = x + 2 (vì x ≥ 2 nên x3 – 3x > 0) ⇔ x6 – 6x4 + 9x2 –                  x –2=0⇔
            5        4          3    2                                 2   3           3
(x – 2).(x + 2x – 2x – 4x + x + 1) = 0 ⇔ (x – 2).(x (x – 4) + 2x (x – 1) + x + 1) = 0 ⇔ x = 2 (do x ≥ 2). V y
limxn = 2.
VD16. Tính gi i h n:
              [(2+ 3) n ]
a)        lim             , trong ñó [x] là ph n nguyên c a s th c x, t c là s nguyên l n nh t không vư t quá x.
               (2+ 3) n
                                                                   c
           1 3       5      2n − 1              n                                             1
b)     lim( + 2 + 3 + ... +    n
                                   ).     c) lim n (a > 1, c > 0).     d)lim n n .    e) lim n .
           2 2       2       2                  a                                             n!
                 (2 + 3) + (2 − 3)
                        n             n
HD. a) ð t a n =                        , nh công th c khai tri n nh th c Niuton ta th y ngay an là s nguyên
                          2
dương. M t khác – 1< – (2 − 3) < 0 nên có [(2+ 3) n ] = [2an – (2 − 3) ] = 2an +[– (2 −                                3)n ] = 2an – 1.
                                             n                                                    n

        [(2+ 3) n ]        2a n − 1       (2 + 3)n + (2 − 3) n − 1                1         1
V y lim             = lim           = lim                          = lim(1 +            −         ) = 1.
         (2+ 3) n         (2+ 3) n              (2+ 3)n                      (7 + 4 3) n (2+ 3) n

Nh n xét. V i x, y, n, r nguyên dương, r không là s chính phương thì t n t i hai dãy s nguyên dương (an) và
(bn)  sao    cho      (x    +y r )n    =    an    + bn r ;   (x    -    y r )n     =   an   - bn r ;    v i
       (x + y r)n + (x − y r)n        (x + y r)n − (x − y r)n
an =                           , bn =                         .
                  2                              2
                             1 3    5        2n − 1        3 5         2n − 1
b) Ta ñ t xn =                + 2 + 3 + ... + n ⇒ 2xn = 1 + + 2 + ... + n-1 . Ta có xn = 2xn – xn =
                             2 2   2          2            2 2          2
        1 1           1    1 − 2n    1      1   n                  1          1
=2+      + 2 + ... + n −2 + n = 3 – n − 2 + n – n −1 . D th y lim n − 2 = lim n = 0. V i m i n > 2
        2 2         2        2     2       2 2                   2           2
                                                     n−1          n 2 − 3n + 2
ta có 2n −1 = Cn−1 + Cn−1 + Cn−1 + ... + Cn−1 ≥ C 2 −1 =                       > 0. T ñó suy ra 0 < n <      2n
                     0         1         2
                                                  n                                                                  , ∀ n > 2.
                                                                                                     n −1
                                                                        2                          2      n − 3n + 2
                                                                                                           2


                2n                               n
Mà lim                         = 0 nên lim n −1 = 0. V y lim xn = 3.
           n 2 − 3n + 2                      2
                                                                                                                           xa.nguyenvan@gmail.com
www.VNMATH.com

TÂM SÁNG – CHÍ B N

                                                     c
                                           
                                           c              c          1
                                       n  n =  n  ñó b = a c – 1 > 0. Nh n th y v i m i n ≥ 2 thì
c) V i hai s a > 1 và c > 0 ta bi n ñ i n = n
                                       a     (1+b) n 
                                            c
                                                       
                                           a
                                            
                                                 (n 2 − n)b 2                  n       2
(1 + b) = Cn + bCn + b Cn + ... + b Cn ≥ b Cn =
       n   0     1    2 2          n n    2 2
                                                              > 0 suy ra 0 <        <         , ∀ n ∈ N *.
                                                      2                      (1+b) (n − 1)b 2
                                                                                  n

                                                                                    c
                    2                    n                    nc        n 
T ñây và do lim            = 0 nên lim        = 0, d n t i lim n = lim          = 0.
                (n − 1)b 2             (1+b)n                 a         (1+b)n 
                                                         n
                                                  1        C1 C2 C3          Cn           n − 1 n 2 − 3n + 2
d)   V i     m i     n   ≥   9   ta   có       1+    = C0 + n + n + n + ... + n ≥ 1 + n +
                                                          n                                      +             =
                                                   n        n n n n           nn            2       6 n
  1 1        n n n          1      n n n           n n n        n( n − 3)                            1
=   +    n+ +          +         > +          =n+       − =n+             ≥ n . D n ñ n 1 < n n < 1+    v i
  2 2        2      6    3 n 2            6          6    2        6                                  n
                        1
m i n ≥ 9. Mà lim (1 +      ) = 1 nên suy ra lim n n = 1.
                         n
                                   log a n
Nh n xét. V i a >0, a ≠ 1, thì lim         = 0.
                                     n
                                                  1    1
e) Trư c h t ta có n! = 1.2.3…n ≤ nn ⇒ ≤                  , ∀ n ∈ N *. M t khác ∀k = 1, n ta luôn có
                                                  n n n!
(n − k)(k − 1) ≥ 0 ⇒ k(n − k + 1) ≥ n , cho k ch y t 1 ñ n n ta thu ñư c n b t ñ ng th c mà hai v ñ u dương, nhân
                                                                                           1     1
n b t ñ ng th c này, v v i v tương ng, ta ñư c (n!)2 ≥ nn hay
                                                                                         n
                                                                                              ≤     , ∀n ∈ ℕ* . T
                                                                                           n!     n
1     1        1                     1         1                    1
   ≤n ≤           , ∀n ∈ ℕ* , và lim = lim         = 0 suy ra lim
                                                                  n
                                                                       = 0.
n      n!       n                    n          n                   n!
                                                      un
VD17. Cho dãy s (un) th a mãn u1 = – 2, un+1 =             , ∀ n ∈ N *.
                                                    1 − un
a. Ch ng minh un < 0, ∀ n ∈ N *.
               1+ u n
b. ð t vn =           , ∀ n ∈ N *. Ch ng minh (vn) là c p s c ng.
                un
c. Tìm công th c s h ng t ng quát c a (un), (vn), và tính các gi i h n limun, limvn.

HD. a) Ta ch ng minh un < 0 (∀ n ∈ N *) b ng phương pháp quy n p toán h c. Rõ ràng u1 = – 2 < 0. Bây gi gi
                                            uk
s uk < 0 ⇒ 1 – uk > 0. D n t i uk+1 =            < 0. V y un < 0 (∀ n ∈ N *).
                                         1 − uk
                1+ u n                                  1                     1− 2 1                  un
b) ð t vn =               thì vn ≠ 1 và un =                 . Ta có v1 =          = . T un+1 =              ta có
                  un                                  vn − 1                   −2    2              1− un
     1        1             1                                                                                   1
          =        : (1 −        ) hay vn+1 = vn – 1 (∀ n ∈ N *). V y (vn) là c p s c ng có s h ng ñ u tiên v1= ,
 v n+1 − 1 v n − 1        vn − 1                                                                                2

công sai d = –1.

                                                                                                  xa.nguyenvan@gmail.com
www.VNMATH.com
TÂM SÁNG – CHÍ B N


                                  3                1       1                  2
c) T câu b ta có ngay vn =          – n, và un =       =          hay un =        (∀ n ∈ N *). Như v y limun = 0,
                                  2              vn − 1 3 − n − 1          1 − 2n
                                                         2
limvn = – ∞.
                                                                                         1
VD18. Cho dãy s (un) th a mãn 0 < un < 1 và un+1 < 1 – 1 v i m i n ∈ N *. Ch ng minh un > , ∀ n ∈ N *.
                                                              4u n                                           2
                                         1                        1
HD. T 0 < un < 1 và un+1 < 1 –               suy ra un(1 – un+1) > (∀ n ∈ N *). Áp d ng b t ñ ng th c Côsi cho hai s
                                        4u n                      4
un và 1 – un+1 ta có un + (1 – un+1) ≥ 2 u n (1 − u n+1 ) > 2. 1 = 1 hay u n + 1 − u n+1 > 1 hay u n > u n+1 (∀ n ∈ N *).
                                                               4
Dãy (un) gi m và b ch n nên có gi i h n h u h n lim u n = u ∈ [0; 1] . L y gi i h n hai v c a b t ñ ng th c
              1                    1         1               1
un(1 – un+1) >   ta ñư c u(1 – u) > ⇔ u = . T c là limun = .
              4                     4        2               2
                                 1
Bây gi ta ñi ch ng minh un >       (∀ n ∈ N *) b ng phương pháp ph n ch ng. Gi s t n t i s nguyên dương k
                                 2
                           1                                                 1                          1
sao cho uk ≤ 1 . Lúc này ≥ uk > uk+1 > … > un+k > … , ∀ n ∈ N *. Suy ra        ≥ uk > uk+1≥ lim un + k = .
              2            2                                                 2              n→+∞        2
                         1
ði u này vô lí. V y un >    (∀ n ∈ N *).
                         2
BÀI T P
9) Tính gi i h n:
                     13 33 53             (2n − 1)3                         13 + 23 + ... + n 3
             1) lim( 3 + 3 + 3 + ... +              ).               2) lim                      .
                     n    n    n              n3                                   n4
                      1     1            1                                   1 3 2n − 1
             3) lim(    +     + ... +          ).                    4) lim( . ...           ).
                     1.2 2.3          n(n + 1)                               2 4      2n
                        1         1                                              1             1                 1
             5) lim(cos + a.sin )n.                                  6) lim(          +               + ... +          ).
                        n         n                                            n2 +1        n2 + n              n2 + n
                      11 + 22 + ... + n n                                    an
             7) lim                       .                          8) lim      (v i a > 0).
                             nn                                              n!
                                                                                   1       1            1
             9) lim     1 + 2n + ... + 2010n .
                      n n
                                                                     10) lim( 1 +     +        + ... +       ).
                                                                                    2       3            n
10) Dãy s (xn) có x1 = a >0, x 2 − x n = a (∀ n ∈ N *). Tìm limxn.
                               n+1

11) Dãy s (an) có 0 < an < 1 và an+1 = an(2 – an) v i m i n ∈ N *. Tìm limxn.

6. DÃY S SINH B I PHƯƠNG TRÌNH
VD19. Cho s nguyên dương n. Ch ng minh r ng phương trình xn+1 = x + 1 có m t nghi m dương duy nh t, kí
hi u là xn. Tìm limxn.
HD. Trư c h t n u x > 0 thì x + 1 > 1, t phương trình xn+1 = x + 1 ⇒ xn+1> 1 ⇒ x > 1. Do ñó n u phương trình
xn+1 = x + 1 có nghi m dương thì nghi m ñó s l n hơn 1. Ta xét hàm s fn(x) = xn+1 – x – 1 v i x > 1, có ñ o hàm
fn’(x) = (n + 1)xn –1 > (n + 1) – 1 = n > 0 v i m i x > 1. Suy ra fn(x) ñ ng bi n trên kho ng (1; + ∞). T ñây, và
tính liên t c c a fn(x), và fn(1) = – 1< 0, lim f n (x) = +∞ , ta k t lu n phương trình xn+1 = x + 1 có nghi m
                                                  x→+∞
dương duy nh t xn. T t nhiên xn > 1. Do fn+1(x) liên t c, fn+1(1) = – 1< 0, fn+1(xn) = x n+2 − x n − 1 > x n − x n − 1 =
                                                                                         n
                                                                                                           n+1


                                                                                                             xa.nguyenvan@gmail.com
www.VNMATH.com
TÂM SÁNG – CHÍ B N


= fn(xn) = 0, nên phương trình xn+2 = x + 1 có nghi m dương duy nh t xn+1 và 1 < xn+1< xn. Dãy (xn) gi m và b
ch n dư i b i 1 nên có gi i h n h u h n limxn = a ≥ 1. Do xn là nghi m dương c a phương trình xn+1 = x + 1 nên
                                       1                                       1
          − 1 = 0 ⇒ x n = (1 + x n ) n +1 ⇒ limx n = lim(1 + x n ) n +1 = (1 + a ) = 1.
                                                                                                0
x n+1 − x n
  n
VD20. Cho n là m t s nguyên dương > 1. Ch ng minh r ng phương trình xn = x + 1 có m t nghi m dương duy
nh t, ký hi u là xn. Ch ng minh r ng xn d n v 1 khi n d n ñ n vô cùng và tìm lim n( x n − 1) .
                                                                                                      n→ ∞
HD. Rõ ràng xn > 1. ð t fn(x) = xn – x – 1. Khi ñó fn+1(1) = - 1 < 0 và fn+1(xn) = xnn+1 – xn – 1 > xnn – xn – 1=
=fn(xn) = 0. T ñó ta suy ra 1 < xn+1 < xn . Suy ra dãy {xn} có gi i h n h u h n a. Ta ch ng minh a = 1. Th t v y,
gi s a > 1. Khi ñó xn ≥ a v i m i n và ta tìm ñư c n ñ l n sao cho: xnn ≥ an > 3 và xn + 1 < 3, mâu thu n vì
fn(xn) = 0. ð gi i ph n cu i c a bài toán, ta ñ t xn = 1 + yn v i lim yn = 0. Thay vào phương trình fn(xn) = 0, ta
ñư c (1+yn)n = 2 + yn. L y logarith hai v , ta ñư c nln(1+yn) = ln(2+yn). T ñó suy ra lim nln(1+yn) = ln2.
Nhưng lim ln(1+yn)/yn = 1 nên t ñây ta suy ra lim nyn = ln2, t c là lim n( xn − 1) = ln 2.
                                                                                       n →∞
                                                           1   1           1
VD21. Ký hi u xn là nghi m c a phương trình:                 +    + ... +     = 0 thu c kho ng (0, 1)
                                                           x x −1         x−n
a) Ch ng minh dãy {xn} h i t
b) Hãy tìm gi i h n ñó.
HD. Rõ ràng xn ñư c xác ñ nh 1 cách duy nh t, 0 < xn < 1. Ta có fn+1(xn) = fn(xn) + 1/(xn-n-1) = 1/(xn-n-1) < 0,
trong khi ñó fn+1(0+) > 0. Theo tính ch t c a hàm liên t c, trên kho ng (0, xn) có ít nh t 1 nghi m c a fn+1(x).
Nghi m ñó chính là xn+1. Như th ta ñã ch ng minh ñư c xn+1 < xn. T c là dãy s {xn} gi m. Do dãy này b ch n
dư i b i 0 nên dãy s có gi i h n. Ta s ch ng minh gi i h n nói trên b ng 0. ð ch ng minh ñi u này, ta c n
ñ n k t qu quen thu c sau: 1 + 1/2 + 1/3 + … + 1/n > ln(n) (Có th ch ng minh d dàng b ng cách s d ng ñánh
giá ln(1+1/n) < 1/n). Th t v y, gi s lim xn = a > 0. Khi ñó, do dãy s gi m nên ta có xn ≥ a v i m i n. Do 1 +
1/2 + 1/3 + … + 1/n +∞ khi n + ∞ nên t n t i N sao cho v i m i n ≥ N ta có 1 + 1/2 + 1/3 + … + 1/n > 1/a.
Khi ñó v i n ≥ N ta có 0 = 1 + 1 + ... + 1 < 1 + 1 + 1 + ... + 1 < 1 − 1 = 0 . Mâu thu n. V y ta
                              xn xn − 1        xn − n xn −1 −2             −n a a
ph i có lim xn = 0.

VD22. (VMO 2007) Cho s th c a > 2 và fn(x) = a10xn+10 + xn + …+x + 1.
a) Ch ng minh v i m i s nguyên dương n, phương trình fn(x) = a luôn có ñúng m t nghi m dương duy nh t.
b) G i nghi m ñó là xn, ch ng minh r ng dãy {xn} có gi i h n h u h n khi n d n ñ n vô cùng.
HD. K t qu c a câu a) là hi n nhiên vì hàm fn(x) tăng trên (0, +∞). D dàng nh n th y 0 < xn < 1. Ta s ch ng
minh dãy xn tăng, t c là xn+1 > xn. Tương t như nh ng l i gi i trên, ta xét
                        fn+1(xn) = a10xnn+11 + xnn+1 + xnn + … + x + 1 = xnfn(xn) + 1 = axn + 1.
                       10
Vì ta ñã có fn+1(1) = a + n + 1 > a nên ta ch c n ch ng minh axn + 1 < a là s suy ra xn < xn+1 < 1. Như v y, c n
ch ng minh xn < (a-1)/a. Th t v y, n u xn ≥ (a-1)/a thì
                                                                n +1
                                                         a −1 
                                            n +10    1−                               n                     n
                                      a −1                                       a −1                a −1 
                                                    + 
                                                            a 
                      fn (xn ) ≥ a10                                 = (a −1)10        + a − (a −1)        >a
                                      a                   a −1                   a                   a 
                                                       1−
                                                              a
(do a – 1 > 1). V y dãy s tăng {xn} tăng và b ch n b i 1 nên h i t .
                                                                                    1     1              1    1
VD23. (VMO 2002) Cho n là m t s nguyên dương. Ch ng minh r ng phương trình             +       + ... + 2    =
                                                                                  x − 1 4x − 1        n x −1 2
có m t nghi m duy nh t xn > 1. Ch ng minh r ng khi n d n ñ n vô cùng, xn d n ñ n 4.

HD. ð t fn(x) như trên và g i xn là nghi m > 1 duy nh t c a phương trình fn(x) = 0. Ta có
            1     1            1   1 1 1            1        1 1 1 1 1 1         1   1 1   1
fn (4) =       +      + ... + 2 − = + + ... +               − =  − + − + ... +      − − =−
           4 −1 16 −1        4n −1 2 1.3 3.5  (2n −1)(2n +1) 2 2  1 3 3 5      2n −1 2n  2 4n
                                                                                                                      xa.nguyenvan@gmail.com
www.VNMATH.com
TÂM SÁNG – CHÍ B N


Áp d ng ñ nh lý Lagrange, ta có : 1/4n = |fn(xn) – f(4)| = |f’(c)||xn-4| v i c thu c kho ng (xn, 4). Nhưng do
                 1            4             1
| f n '(c) |=           +            + ... > n ên t ñây |xn – 4| < 9/4n, suy ra lim xn = 4.
              (c − 1) 2
                          (4c − 1) 2        9
VD24. Cho n là m t s nguyên dương > 1. Ch ng minh r ng phương trình xn = x2 + x + 1 có m t nghi m dương
duy nh t, ký hi u là xn. Hãy tìm s th c a sao cho gi i h n lim n ( x n − x n +1 ) t n t i, h u h n và khác 0.
                                                                         a
                                                                   n→∞
HD. ð t Pn(x) = xn – x2 – x – 1. Ta có Pn+1(x) = xn+1 – x2 – x – 1 = xn+1 – xn + Pn(x) = xn(x-1) + Pn(x).
T ñó Pn+1(xn) = xnn(xn-1) + Pn(xn) = (xn2+xn+1)(xn-1) = xn3 – 1.
Áp d ng ñ nh lý Lagrange, ta có: (xn2+xn+1)(xn – 1) = Pn+1(xn) – Pn+1(xn+1) = (xn – xn+1)Pn+1’(c)
v i c thu c (xn+1, xn), Pn+1’(x) = (n+1)xn – 2x – 1. T ñó
(n+1)(xn+1+1+1/xn+1) – 2xn+1 – 1 = Pn+1’(xn+1) < Pn+1’(c) < Pn+1’(xn)= (n+1)(xn2+xn+1) – 2xn – 1.
T ñây, v i lưu ý lim xn = 1, ta suy ra : lim Pn +1 '(c ) = 3 .Ti p t c s d ng lim n(xn – 1) = 3, ta suy ra:
                                               n →∞      n
 lim nPn +1 (c)( xn − xn +1 ) = lim n( xn + xn + 1)( xn − 1) = 3ln(3)
        '                               2
n →∞                            n →∞

                            Pn' +1 (c)                                       P ' (c )
⇔ lim n 2 ( xn − xn +1 ).              = 3ln(3) ⇔ lim n 2 ( xn − xn +1 ) lim n +1 = 3ln(3)
    n →∞                         n                n →∞                  n →∞    n
⇔ lim n 2 ( xn − xn +1 )3 = 3ln(3) ⇔ lim n 2 ( xn − xn +1 ) = ln(3)
    n →∞                                 n →∞
V y v i c = 2 thì gi i h n ñã cho t n t i, h u h n và khác 0. D th y v i c > 2 thì gi i h n ñã cho b ng vô cùng
và n i c < 2 thì gi i h n ñã cho b ng 0. V y c = 2 là ñáp s duy nh t c a bài toán.

BÀI T P
12) V i m i s nguyên dương n phương trình x =               n
                                                                x + 1 có m t nghi m dương duy nh t, kí hi u là xn. Tìm
lim(n(xn – 1)).




                                                                                                         xa.nguyenvan@gmail.com

More Related Content

What's hot

Bài tập đạo hàm có hướng dẫn
Bài tập đạo hàm có hướng dẫnBài tập đạo hàm có hướng dẫn
Bài tập đạo hàm có hướng dẫndiemthic3
 
Phân dạng đề thi tốt nghiệp truonghocso.com
Phân dạng đề thi tốt nghiệp   truonghocso.comPhân dạng đề thi tốt nghiệp   truonghocso.com
Phân dạng đề thi tốt nghiệp truonghocso.comThế Giới Tinh Hoa
 
Tichphan mathvn.com-transitung
Tichphan mathvn.com-transitungTichphan mathvn.com-transitung
Tichphan mathvn.com-transitungQuyen Le
 
Biện luận phương trình chứa dấu giá trị tuyệt đối
Biện luận phương trình chứa dấu giá trị tuyệt đốiBiện luận phương trình chứa dấu giá trị tuyệt đối
Biện luận phương trình chứa dấu giá trị tuyệt đốiThopeo Kool
 
OT HK II - 11
OT HK II - 11OT HK II - 11
OT HK II - 11Uant Tran
 
Da toan-chi-tiet-b 2010
Da toan-chi-tiet-b 2010Da toan-chi-tiet-b 2010
Da toan-chi-tiet-b 2010nhathung
 
đạO hàm và vi phân
đạO hàm và vi phânđạO hàm và vi phân
đạO hàm và vi phânchuateonline
 
10 dạng tích phân thường gặp thanh tùng
10 dạng tích phân thường gặp   thanh tùng10 dạng tích phân thường gặp   thanh tùng
10 dạng tích phân thường gặp thanh tùngTrần Hà
 
Chuyên đề luyện thi Đại học 2014
Chuyên đề luyện thi Đại học 2014Chuyên đề luyện thi Đại học 2014
Chuyên đề luyện thi Đại học 2014tuituhoc
 
Tích phân hàm phân thức luyện thi đại học
Tích phân hàm phân thức luyện thi đại họcTích phân hàm phân thức luyện thi đại học
Tích phân hàm phân thức luyện thi đại họcGia sư Đức Trí
 
ôN thi tốt nghiep thpt-montoan-theo dan gbai - truonghocso.com
ôN thi tốt nghiep thpt-montoan-theo dan gbai - truonghocso.comôN thi tốt nghiep thpt-montoan-theo dan gbai - truonghocso.com
ôN thi tốt nghiep thpt-montoan-theo dan gbai - truonghocso.comThế Giới Tinh Hoa
 
Tích phân hàm phân thức hữu tỷ (part 1)
Tích phân hàm phân thức hữu tỷ (part 1)Tích phân hàm phân thức hữu tỷ (part 1)
Tích phân hàm phân thức hữu tỷ (part 1)Oanh MJ
 
Bai giang nguyen ham tich phan 2015 q1
Bai giang nguyen ham   tich phan 2015 q1Bai giang nguyen ham   tich phan 2015 q1
Bai giang nguyen ham tich phan 2015 q1Hien Nguyen
 

What's hot (17)

Bài tập đạo hàm có hướng dẫn
Bài tập đạo hàm có hướng dẫnBài tập đạo hàm có hướng dẫn
Bài tập đạo hàm có hướng dẫn
 
Phân dạng đề thi tốt nghiệp truonghocso.com
Phân dạng đề thi tốt nghiệp   truonghocso.comPhân dạng đề thi tốt nghiệp   truonghocso.com
Phân dạng đề thi tốt nghiệp truonghocso.com
 
Tichphan mathvn.com-transitung
Tichphan mathvn.com-transitungTichphan mathvn.com-transitung
Tichphan mathvn.com-transitung
 
Biện luận phương trình chứa dấu giá trị tuyệt đối
Biện luận phương trình chứa dấu giá trị tuyệt đốiBiện luận phương trình chứa dấu giá trị tuyệt đối
Biện luận phương trình chứa dấu giá trị tuyệt đối
 
OT HK II - 11
OT HK II - 11OT HK II - 11
OT HK II - 11
 
Da toan-chi-tiet-b 2010
Da toan-chi-tiet-b 2010Da toan-chi-tiet-b 2010
Da toan-chi-tiet-b 2010
 
200 cau-khaosathamso2 (1) 06
200 cau-khaosathamso2 (1) 06200 cau-khaosathamso2 (1) 06
200 cau-khaosathamso2 (1) 06
 
đạO hàm và vi phân
đạO hàm và vi phânđạO hàm và vi phân
đạO hàm và vi phân
 
10 dạng tích phân thường gặp thanh tùng
10 dạng tích phân thường gặp   thanh tùng10 dạng tích phân thường gặp   thanh tùng
10 dạng tích phân thường gặp thanh tùng
 
Chuyên đề luyện thi Đại học 2014
Chuyên đề luyện thi Đại học 2014Chuyên đề luyện thi Đại học 2014
Chuyên đề luyện thi Đại học 2014
 
Tích phân hàm phân thức luyện thi đại học
Tích phân hàm phân thức luyện thi đại họcTích phân hàm phân thức luyện thi đại học
Tích phân hàm phân thức luyện thi đại học
 
100 cau hoi phu kshs
100 cau hoi phu kshs100 cau hoi phu kshs
100 cau hoi phu kshs
 
ôN thi tốt nghiep thpt-montoan-theo dan gbai - truonghocso.com
ôN thi tốt nghiep thpt-montoan-theo dan gbai - truonghocso.comôN thi tốt nghiep thpt-montoan-theo dan gbai - truonghocso.com
ôN thi tốt nghiep thpt-montoan-theo dan gbai - truonghocso.com
 
Tích phân hàm phân thức hữu tỷ (part 1)
Tích phân hàm phân thức hữu tỷ (part 1)Tích phân hàm phân thức hữu tỷ (part 1)
Tích phân hàm phân thức hữu tỷ (part 1)
 
Bai giang nguyen ham tich phan 2015 q1
Bai giang nguyen ham   tich phan 2015 q1Bai giang nguyen ham   tich phan 2015 q1
Bai giang nguyen ham tich phan 2015 q1
 
Basic số phức cực hay
Basic số phức cực hayBasic số phức cực hay
Basic số phức cực hay
 
Bbt
BbtBbt
Bbt
 

Similar to Chuyến đề dãy số

9dethithu
9dethithu9dethithu
9dethithuDuy Duy
 
Dc ôn tâp hkii
Dc ôn tâp hkiiDc ôn tâp hkii
Dc ôn tâp hkiintquangbs
 
Dc ôn tâp hkii
Dc ôn tâp hkiiDc ôn tâp hkii
Dc ôn tâp hkiintquangbs
 
De thi hoc ki 2 k12 nam 0910
De thi hoc ki 2 k12 nam 0910De thi hoc ki 2 k12 nam 0910
De thi hoc ki 2 k12 nam 0910lvquy
 
De Thi Hoc Ki 2 K12 Nam 0910
De Thi Hoc Ki 2 K12 Nam 0910De Thi Hoc Ki 2 K12 Nam 0910
De Thi Hoc Ki 2 K12 Nam 0910lvquy
 
De thi hoc ki 2 k12 nam 0910
De thi hoc ki 2 k12 nam 0910De thi hoc ki 2 k12 nam 0910
De thi hoc ki 2 k12 nam 0910lvquy
 
De thi hoc ki 2 k12 nam 0910
De thi hoc ki 2 k12 nam 0910De thi hoc ki 2 k12 nam 0910
De thi hoc ki 2 k12 nam 0910lvquy
 
Toanb2011
Toanb2011Toanb2011
Toanb2011Duy Duy
 
Deontoanso6
Deontoanso6Deontoanso6
Deontoanso6Duy Duy
 
Www.mathvn.com day-so-nguyen
Www.mathvn.com day-so-nguyen Www.mathvn.com day-so-nguyen
Www.mathvn.com day-so-nguyen makiemcachthe
 
Phongmath day-so-nguyen tatthu
Phongmath day-so-nguyen tatthuPhongmath day-so-nguyen tatthu
Phongmath day-so-nguyen tatthuphongmathbmt
 
So phuc va cac bai toan lien quan 2
So phuc va cac bai toan lien quan 2So phuc va cac bai toan lien quan 2
So phuc va cac bai toan lien quan 2Huynh ICT
 
Toan totnghiep thpt
Toan totnghiep thptToan totnghiep thpt
Toan totnghiep thptDuy Duy
 
Thi thử toán chuyên nguyễn quang diêu đt 2012 lần 2 k ab
Thi thử toán chuyên nguyễn quang diêu đt 2012 lần 2 k abThi thử toán chuyên nguyễn quang diêu đt 2012 lần 2 k ab
Thi thử toán chuyên nguyễn quang diêu đt 2012 lần 2 k abThế Giới Tinh Hoa
 
Dc ôn tâp hkii
Dc ôn tâp hkiiDc ôn tâp hkii
Dc ôn tâp hkiintquangbs
 
48 de luyen thi dai hoc
48 de luyen thi dai hoc48 de luyen thi dai hoc
48 de luyen thi dai hocDuy Duy
 
48 de luyen thi dai hoc
48 de luyen thi dai hoc48 de luyen thi dai hoc
48 de luyen thi dai hocDuy Duy
 
De thi hoc ki i toan 12 co loi giai 2010-2011 - truonghocso.com
 De thi hoc ki i toan 12 co loi giai 2010-2011 - truonghocso.com De thi hoc ki i toan 12 co loi giai 2010-2011 - truonghocso.com
De thi hoc ki i toan 12 co loi giai 2010-2011 - truonghocso.comThế Giới Tinh Hoa
 

Similar to Chuyến đề dãy số (20)

đạI số tổ hợp chương 2
đạI số tổ hợp chương 2đạI số tổ hợp chương 2
đạI số tổ hợp chương 2
 
9dethithu
9dethithu9dethithu
9dethithu
 
Dc ôn tâp hkii
Dc ôn tâp hkiiDc ôn tâp hkii
Dc ôn tâp hkii
 
Dc ôn tâp hkii
Dc ôn tâp hkiiDc ôn tâp hkii
Dc ôn tâp hkii
 
De thi hoc ki 2 k12 nam 0910
De thi hoc ki 2 k12 nam 0910De thi hoc ki 2 k12 nam 0910
De thi hoc ki 2 k12 nam 0910
 
De Thi Hoc Ki 2 K12 Nam 0910
De Thi Hoc Ki 2 K12 Nam 0910De Thi Hoc Ki 2 K12 Nam 0910
De Thi Hoc Ki 2 K12 Nam 0910
 
De thi hoc ki 2 k12 nam 0910
De thi hoc ki 2 k12 nam 0910De thi hoc ki 2 k12 nam 0910
De thi hoc ki 2 k12 nam 0910
 
De thi hoc ki 2 k12 nam 0910
De thi hoc ki 2 k12 nam 0910De thi hoc ki 2 k12 nam 0910
De thi hoc ki 2 k12 nam 0910
 
Toanb2011
Toanb2011Toanb2011
Toanb2011
 
Deontoanso6
Deontoanso6Deontoanso6
Deontoanso6
 
đạI số tổ hợp chương 3
đạI số tổ hợp chương 3đạI số tổ hợp chương 3
đạI số tổ hợp chương 3
 
Www.mathvn.com day-so-nguyen
Www.mathvn.com day-so-nguyen Www.mathvn.com day-so-nguyen
Www.mathvn.com day-so-nguyen
 
Phongmath day-so-nguyen tatthu
Phongmath day-so-nguyen tatthuPhongmath day-so-nguyen tatthu
Phongmath day-so-nguyen tatthu
 
So phuc va cac bai toan lien quan 2
So phuc va cac bai toan lien quan 2So phuc va cac bai toan lien quan 2
So phuc va cac bai toan lien quan 2
 
Toan totnghiep thpt
Toan totnghiep thptToan totnghiep thpt
Toan totnghiep thpt
 
Thi thử toán chuyên nguyễn quang diêu đt 2012 lần 2 k ab
Thi thử toán chuyên nguyễn quang diêu đt 2012 lần 2 k abThi thử toán chuyên nguyễn quang diêu đt 2012 lần 2 k ab
Thi thử toán chuyên nguyễn quang diêu đt 2012 lần 2 k ab
 
Dc ôn tâp hkii
Dc ôn tâp hkiiDc ôn tâp hkii
Dc ôn tâp hkii
 
48 de luyen thi dai hoc
48 de luyen thi dai hoc48 de luyen thi dai hoc
48 de luyen thi dai hoc
 
48 de luyen thi dai hoc
48 de luyen thi dai hoc48 de luyen thi dai hoc
48 de luyen thi dai hoc
 
De thi hoc ki i toan 12 co loi giai 2010-2011 - truonghocso.com
 De thi hoc ki i toan 12 co loi giai 2010-2011 - truonghocso.com De thi hoc ki i toan 12 co loi giai 2010-2011 - truonghocso.com
De thi hoc ki i toan 12 co loi giai 2010-2011 - truonghocso.com
 

More from Thế Giới Tinh Hoa

Cách chụp ảnh công ty đẹp 2019
Cách chụp ảnh công ty đẹp 2019Cách chụp ảnh công ty đẹp 2019
Cách chụp ảnh công ty đẹp 2019Thế Giới Tinh Hoa
 
Bảng báo giá sản phẩm rèm bạch dương
Bảng báo giá sản phẩm rèm bạch dươngBảng báo giá sản phẩm rèm bạch dương
Bảng báo giá sản phẩm rèm bạch dươngThế Giới Tinh Hoa
 
Album sổ mẫu Rèm cửa Bạch Dương
Album sổ mẫu Rèm cửa Bạch DươngAlbum sổ mẫu Rèm cửa Bạch Dương
Album sổ mẫu Rèm cửa Bạch DươngThế Giới Tinh Hoa
 
Cách tắm cho bé vào mùa đông
Cách tắm cho bé vào mùa đôngCách tắm cho bé vào mùa đông
Cách tắm cho bé vào mùa đôngThế Giới Tinh Hoa
 
Giáo trình tự học illustrator cs6
Giáo trình tự học illustrator cs6  Giáo trình tự học illustrator cs6
Giáo trình tự học illustrator cs6 Thế Giới Tinh Hoa
 
Nữ quái sân trườngtruonghocso.com
Nữ quái sân trườngtruonghocso.comNữ quái sân trườngtruonghocso.com
Nữ quái sân trườngtruonghocso.comThế Giới Tinh Hoa
 
Những chàng trai xấu tính nguyễn nhật ánhtruonghocso.com
Những chàng trai xấu tính  nguyễn nhật ánhtruonghocso.comNhững chàng trai xấu tính  nguyễn nhật ánhtruonghocso.com
Những chàng trai xấu tính nguyễn nhật ánhtruonghocso.comThế Giới Tinh Hoa
 
Những bài văn mẫu dành cho học sinh lớp 10truonghocso.com
Những bài văn mẫu dành cho học sinh lớp 10truonghocso.comNhững bài văn mẫu dành cho học sinh lớp 10truonghocso.com
Những bài văn mẫu dành cho học sinh lớp 10truonghocso.comThế Giới Tinh Hoa
 

More from Thế Giới Tinh Hoa (20)

Cách chụp ảnh công ty đẹp 2019
Cách chụp ảnh công ty đẹp 2019Cách chụp ảnh công ty đẹp 2019
Cách chụp ảnh công ty đẹp 2019
 
Lỗi web bachawater
Lỗi web bachawaterLỗi web bachawater
Lỗi web bachawater
 
Bảng báo giá sản phẩm rèm bạch dương
Bảng báo giá sản phẩm rèm bạch dươngBảng báo giá sản phẩm rèm bạch dương
Bảng báo giá sản phẩm rèm bạch dương
 
Album sổ mẫu Rèm cửa Bạch Dương
Album sổ mẫu Rèm cửa Bạch DươngAlbum sổ mẫu Rèm cửa Bạch Dương
Album sổ mẫu Rèm cửa Bạch Dương
 
thong tin lam viec tren lamchame
thong tin lam viec tren lamchamethong tin lam viec tren lamchame
thong tin lam viec tren lamchame
 
Cách tắm cho bé vào mùa đông
Cách tắm cho bé vào mùa đôngCách tắm cho bé vào mùa đông
Cách tắm cho bé vào mùa đông
 
Giáo trình tự học illustrator cs6
Giáo trình tự học illustrator cs6  Giáo trình tự học illustrator cs6
Giáo trình tự học illustrator cs6
 
Nang luc truyen thong
Nang luc truyen thongNang luc truyen thong
Nang luc truyen thong
 
Huongdansudung izishop
Huongdansudung izishopHuongdansudung izishop
Huongdansudung izishop
 
Ho so nang luc cong ty
Ho so nang luc cong tyHo so nang luc cong ty
Ho so nang luc cong ty
 
seo contract
seo contractseo contract
seo contract
 
di google cong
di google congdi google cong
di google cong
 
E1 f4 bộ binh
E1 f4 bộ binhE1 f4 bộ binh
E1 f4 bộ binh
 
E2 f2 bộ binh
E2 f2 bộ binhE2 f2 bộ binh
E2 f2 bộ binh
 
E3 f1 bộ binh
E3 f1 bộ binhE3 f1 bộ binh
E3 f1 bộ binh
 
E2 f1 bộ binh
E2 f1 bộ binhE2 f1 bộ binh
E2 f1 bộ binh
 
E1 f1 bộ binh
E1 f1 bộ binhE1 f1 bộ binh
E1 f1 bộ binh
 
Nữ quái sân trườngtruonghocso.com
Nữ quái sân trườngtruonghocso.comNữ quái sân trườngtruonghocso.com
Nữ quái sân trườngtruonghocso.com
 
Những chàng trai xấu tính nguyễn nhật ánhtruonghocso.com
Những chàng trai xấu tính  nguyễn nhật ánhtruonghocso.comNhững chàng trai xấu tính  nguyễn nhật ánhtruonghocso.com
Những chàng trai xấu tính nguyễn nhật ánhtruonghocso.com
 
Những bài văn mẫu dành cho học sinh lớp 10truonghocso.com
Những bài văn mẫu dành cho học sinh lớp 10truonghocso.comNhững bài văn mẫu dành cho học sinh lớp 10truonghocso.com
Những bài văn mẫu dành cho học sinh lớp 10truonghocso.com
 

Chuyến đề dãy số

  • 1. www.VNMATH.com CHUYÊN ð DÃY S (BDHSG) 1. KHÁI NI M DÃY S 1) Cho A là m t t p con khác r ng c a t p s nguyên ℤ, hàm s u:A → ℝ n ֏ u(n) = u n ñư c g i là m t dãy s , và kí hi u là (u n ) ho c {u n } . Thông thư ng ta hay ch n A sao cho ph n t nh nh t c a A là 1. Dãy (un) g i là dãy s h u h n (ho c dãy s vô h n) n u A là t p h p g m h u h n (vô h n) ph n t . S un ñư c g i là s h ng t ng quát c a dãy (un). 2) Dãy s (un) ñư c g i là dãy s tăng (tăng không nghi m ng t, gi m, gi m không nghiêm ng t) n u u n < u n +1 (tương ng u n ≤ u n +1 , u n > u n +1 , u n ≥ u n +1 ) v i m i n ∈ A. 3) Dãy s (un) ñư c g i là tu n hoàn n u t n t i s nguyên dương k sao cho u n + k = u n , ∀n ∈ A. S k nh nh t tho mãn tính ch t này ñư c g i là chu kì c a dãy tu n hoàn (un). N u k = 1 thì ta ñư c m t dãy h ng (t t c các s h ng b ng nhau). 4) Dãy s (un) ñư c g i là b ch n trên n u t n t i s th c M sao cho u n ≤ M v i m i n ∈ A. Dãy s (un) ñư c g i là b ch n dư i n u t n t i s th c m sao cho u n ≥ m v i m i n ∈ A. Dãy s (un) ñư c g i là b ch n (ho c gi i n i) n u nó v a b ch n trên v a b ch n dư i, t c là t n t i s th c M, m sao cho m ≤ u n ≤ M v i m i n ∈ A, ho c t n t i s th c C sao cho u n ≤ C, ∀n ∈ A. Dãy s h u h n ho c tu n hoàn thì luôn b ch n. 2. C P S 1) C p s c ng - Dãy s (un) ñư c g i là c p s c ng n u m i s h ng ñ u tho mãn u n +1 − u n = d (d: h ng s , g i là công sai). - Công th c truy h i: u n +1 = u n + d. Công th c s h ng t ng quát: u n = u1 + (n − 1)d, ∀n ∈ A. Công th c tính n n n(n − 1) t ng n s h ng ñ u tiên: Sn = (u1 + u n ) = (2u1 + (n − 1)d) = nu1 + d. Tính ch t các s h ng: 2 2 2 u k +1 + u k −1 = 2u k . 2) C p s nhân - Dãy s (un) ñư c g i là c p s nhân n u m i s h ng ñ u tho mãn u n +1 = u n .q (q: h ng s , g i là công b i). - Công th c truy h i: u n +1 = u n .q. Công th c s h ng t ng quát: u n = u1.q n −1. Công th c tính t ng n s h ng 1 − q n +1 2 ñ u tiên: Sn = nu1 n u q = 1, Sn = u1 n u q ≠ 1. Tính ch t các s h ng: u k +1.u k −1 = u k . 1− q 3) C p s nhân c ng - Dãy s (un) ñư c g i là c p s nhân c ng n u m i s h ng ñ u tho mãn u n +1 = u n .q + d (q, d là h ng s ). 4) C p s ñi u hoà 2u n −1u n +1 - Dãy s (un) ñư c g i là c p s ñi u hoà n u m i s h ng c a nó ñ u khác 0 và tho mãn u n = , u n −1 + u n +1 1 1 1 1 hay = ( + ). (H c sinh t ôn t p các d ng toán v c p s ) u n 2 u n −1 u n +1 3. XÁC ð NH S H NG T NG QUÁT C A DÃY S 3.1. D ðOÁN S H NG T NG QUÁT VÀ CH NG MINH B NG QUI N P BÀI T P 1) Xác ñ nh s h ng t ng quát c a dãy s cho b i: u a)u1 = 1, u n +1 = n , ∀n = 1, 2,3,... b)u1 = 2, u n +1 = 2 + u n , ∀n = 1, 2,3,... 1+ un 3 2 5 3 −1 3 − 1 + ( 3 + 1)u n c)u1 = 1, u n +1 = − u n + u n + 1, ∀n = 1, 2,3,... d)u1 = , u n +1 = , ∀n = 1, 2,3,... 2 2 3 +1 3 + 1 − ( 3 − 1)u n xa.nguyenvan@gmail.com
  • 2. www.VNMATH.com TÂM SÁNG – CHÍ B N 1 1 e)u1 = , u n +1 = 2u n − 1, ∀n = 1, 2,3,... 2 f )u1 = , u n +1 = 2u n 1 − u n , ∀n = 1, 2,3,... 2 2 2 3.2. M T S D NG TRUY H I ð C BI T V i dãy s cho b i công th c truy h i d ng u n +1 = u n + f (n) thì u n = u1 + f (1) + f (2) + ... + f (n − 1). V i dãy s cho b i công th c truy h i d ng u n +1 = u n .g(n) thì u n = u1.g(1).g(2)...g(n − 1). BÀI T P 2) Xác ñ nh s h ng t ng quát c a dãy s cho b i: (n + 1) 2 u n a)u1 = 1, u n +1 = u n + n!.n, ∀n = 1, 2,3,... b)u1 = 1, u n +1 = , ∀n = 1, 2,3,... n(n + 2) c)u1 = 1, u n +1 = u n + n 3 + 3n 2 − 3n + 1, ∀n = 1, 2,3,... d)u1 = 3, u n +1 = u n + 3n , ∀n = 1, 2,3,... 1 e)u1 = 1, u n +1 = u n + (n + 1).2n , ∀n = 1, 2,3,... f )u1 = 2, u n +1 = 2 − , ∀n = 1, 2,3,... un n2 −1 n g)u1 = 1, u n +1 = u n , ∀n = 1, 2,3,... h)u1 = 0, u n +1 = (1 + u n ), ∀n = 1, 2,3,... n n +1 3.3. PHƯƠNG TRÌNH ð C TRƯNG Ta ch xét hai trư ng h p ñơn gi n sau ñây: a) Xét dãy s (un) cho b i u1, u 2 , u n + 2 = a.u n +1 + b.u n , ∀n ∈ ℕ * (a, b = const). Khi ñó phương trình x 2 − ax − b = 0 ñư c g i là phương trình ñ c trưng c a dãy s ñã cho. N u phương trình trên có hai nghi m th c phân bi t x1, x 2 thì u n = A.x1 + B.x n . n 2 N u phương trình trên có hai nghi m th c trùng nhau x1 = x 2 thì u n = (A + nB).x1 . n N u phương trình trên có ∆ < 0 , g i hai nghi m ph c c a nó là x1 , x 2 , và bi u di n hai s ph c này d ng lư ng giác x1 = r(cos ϕ + i.sin ϕ), x 2 = r(cos ϕ − i.sin ϕ), v i r, ϕ là các s th c, r là môñun c a x1 và x 2 , ϕ ∈ [ 0; 2π ) , i là ñơn v o, thì u n = r n (A.cos nϕ + B.sinnϕ). ( ñó các h ng s A, B ñư c xác ñ nh nh u1, u 2 ) b) Xét dãy s (un) cho b i u1, u 2 , u 3 , u n +3 = a.u n + 2 + b.u n +1 + c.u n , ∀n ∈ ℕ * (a, b, c = const) có phương trình ñ c trưng x 3 − ax 2 − bx − c = 0. N u phương trình trên có ba nghi m th c phân bi t x1, x 2 , x 3 thì u n = A.x1 + B.x n + C.x 3 . n 2 n N u phương trình trên có ba nghi m th c x1 , x 2 , x 3 mà x1 ≠ x 2 = x 3 thì u n = A.x1 + (B + nC).x 2 . n n N u phương trình trên có ba nghi m th c x1 , x 2 , x 3 và x1 = x 2 = x 3 thì u n = (A + nB + n 2C).x1 . n N u phương trình trên có ba nghi m x1, x 2 , x 3 trong ñó x1 là nghi m th c, còn hai nghi m x 2 = r(cos ϕ + i.sin ϕ), x 3 = r(cos ϕ − i.sin ϕ) là hai nghi m ph c (không ph i là s th c) thì u n = A.x1 + r n (B.cos nϕ + C.sinnϕ). ( ñó các h ng s A, B, C ñư c xác ñ nh nh u1, u 2 , u 3 ) n VD1. Cho dãy s (u n ) xác ñ nh b i u1 = 1, u 2 = 0, u n + 2 = u n +1 − u n , ∀n ∈ ℕ *. Ch ng minh (u n ) b ch n. π π HD. Phương trình ñ c trưng c a dãy s ñã cho là x 2 − x + 1 = 0 có hai nghi m ph c x1 = cos + i.sin , 3 3 π π nπ nπ x 2 = cos − i.sin , nên u n = 1n (A.cos + B.sin ), ∀n ∈ ℕ *. Do u1 = 1, u 2 = 0, nên ta có 3 3 3 3  π π A B 3 1 = A.cos 3 + B.sin 3 (= u1 )  + =1 A = 1  2 2   ⇔ ⇔ 3. 0 = A.cos 2π + B.sin 2π (= u 2 ) − A + B 3 = 0 B =    2  3 3 3  2 xa.nguyenvan@gmail.com
  • 3. www.VNMATH.com TÂM SÁNG – CHÍ B N nπ 3 nπ nπ 3 nπ 3 2 Suy ra u n = cos + .sin , ∀n ∈ ℕ *. V y u n = cos + .sin ≤ 12 + ( ) 2 = , ∀n ∈ ℕ*, hay 3 3 3 3 3 3 3 3 (u n ) là dãy b ch n. VD2. Cho (u n ) có u1 = 0, u 2 = 16, u3 = 47, u n +3 = 7u n +2 − 11u n +1 + 5u n , ∀n ∈ ℕ *. Tìm dư khi chia u 2011 cho 2011. HD. Phương trình ñ c trưng x 3 − 7x 2 + 11x − 5 = 0 có 3 nghi m th c x1 = 5 (nghi m ñơn), x 2 = x 3 = 1 (nghi m 1 kép) do ñó u n = A.5n + (B + nC).1n , ∀n ∈ ℕ *. Vì u1 = 0, u 2 = 16, u 3 = 47 nên A = , B = −13, C = 12. Suy ra 5 n −1 u n = 5 + 12n − 13, ∀n ∈ ℕ *. T ñó u 2011 = 5 2010 + 12.2011 − 13. Theo ñ nh lí Phécma thì 52010 − 1⋮ 2011 (ñ nh lí Phécma: N u p là s nguyên t , a là s nguyên và (a, p) = 1, thì a p−1 ≡ 1 (mod p)). V y u 2011 chia cho 2011 dư −12 (hay dư 1999). VD3. Cho hai dãy s (x n ), (y n ) tho mãn x1 = y1 = 1, x n +1 = 4x n − 2y n , y n +1 = x n + y n , ∀n ∈ ℕ *. Xác ñ nh công th c c a x n , y n . HD. Ta có x n + 2 = 4x n +1 − 2y n +1 = 4x n +1 − 2(x n + y n ) = 4x n +1 − 2x n − 2y n = 4x n +1 − 2x n + x n +1 − 4x n hay x n + 2 = 5x n +1 − 6x n (∀n ∈ ℕ*). Dãy (u n ) có phương trình ñ c trưng x 2 − 5x + 6 = 0 ⇔ x = 2 ho c x = 3. Suy ra 1 x n = A.2n + B.3n , ∀n ∈ ℕ *. Mà x1 = 1, x 2 = 4x1 − 2y1 = 2, nên A = , B = 0, và ta có x n = 2n −1, ∀n ∈ ℕ *. T 2 n −1 n −1 ñó và x n +1 = 4x n − 2y n ⇒ y n = 2 . V y x n = y n = 2 , ∀n ∈ ℕ *. 3.4. PHƯƠNG PHÁP DÃY S PH VD4. Cho dãy s (u n ) : u1 = 1, u 2 = 2, u n + 2 = 2.u n +1 − u n + 1, ∀n ∈ ℕ *. ð t v n = u n +1 − u n . Ch ng minh (v n ) là c p s c ng và tìm u n . HD. a) Ta có v1 = u 2 − u1 = 1. Và u n +2 = 2.u n +1 − u n + 1, ∀n ∈ ℕ* ⇔ u n + 2 − u n +1 = u n +1 − u n + 1, ∀n ∈ ℕ * ⇔ v n +1 = v n + 1, ∀n ∈ ℕ *. V y (v n ) là c p s c ng v i s h ng ñ u tiên v1 = 1, công sai d = 1. b) T câu a ta có vn = v1 + (n −1).d = 1+ (n −1).1 = n, hay un+1 − un = n, ∀n ∈ ℕ *. Suy ra un = (un − un−1) + (un−1 − un−2 ) + (n − 1).n n 2 n +... + (u 2 − u1 ) + u1 = [(n − 1) + (n − 2) + ... + 2 + 1] + 1 = 1 + = − + 1, ∀n ∈ ℕ *. 2 2 2 2 1 VD5. Cho dãy s (u n ) : u1 = 0, u 2 = 1, u n + 2 = .u n +1 + u n , ∀n ∈ ℕ *. ð t v n = u n +1 − u n . Ch ng minh (v n ) là 3 3 c p s nhân và tìm u n . 2 1 HD. Ta có v1 = u 2 − u1 = 1. Và u n + 2 = .u n +1 + u n , ∀n ∈ ℕ* ⇔ 3u n + 2 = 2u n +1 + u n , ∀n ∈ ℕ* ⇔ 3 3 −1 3(u n + 2 − u n +1 ) = −(u n +1 − u n ), ∀n ∈ ℕ * ⇔ v n +1 = v n , ∀n ∈ ℕ *. V y (v n ) là c p s nhân v i s h ng ñ u tiên 3 1 v1 = 1, công b i q = − . 3 1 Ta có vn = v1.qn−1 = (− )n−1, ∀n ∈ℕ *. Suy ra un = (un − un−1) + (un−1 − un−2) ++... + (u2 − u1) + u1 = vn−1 + vn−2 + 3 1 1 − (− ) n −1 1 1 1 3 9 +... + v 2 + v1 + u1 = 0 + 1 + (− ) + (− ) 2 + ... + (− ) n − 2 = 1. 3 = + , ∀n ∈ ℕ *. 3 3 3 1 − (− ) 1 4 4.(−3) n 3 VD6. Tìm s h ng t ng quát c a dãy s ( un ) có u1 = c, un+1 = q.un + an + d , ∀n ∈ ℕ*, ñó a, c, d, q là h ng s . xa.nguyenvan@gmail.com
  • 4. www.VNMATH.com TÂM SÁNG – CHÍ B N HD. V i q = 1 thì un+1 = un + an + d , ∀n ∈ ℕ * . Ta nh n th y u 2 = u1 + a + d, u 3 = u 2 + 2a + d,..., un−1 = un −2 + (n − 2)a + d,un = un−1 + (n −1)a + d ⇒ u2 + u3 + ... + un−1 + un = u1 + u2 + ... + un−2 + un−1 + n(n −1) n(n −1) +a(1 + 2 + ... + (n − 2) + (n −1)) + (n −1)d ⇒ un = u1 + a + (n −1)d ⇒ un = c + a + (n −1)d, (n ∈ ℕ*). 2 2 Khi q ≠ 1, ta s xét m t dãy ph ( vn ) th a mãn un = vn + bn + e, n ∈ ℕ*, ñó b, e là nh ng h ng s , và ta c g ng ch n b, e thích h p ñ ( vn ) là c p s nhân. T ñ ng th c truy h i ban ñ u ta có vn +1 = qvn + (qb + a − b)n + (qe + d − b − e), n ∈ ℕ*, và d th y ñ ( vn ) là c p s nhân thì c n có qb + a − b = qe + d − b − e = 0 ⇔ b = a , e = d − a − qd ,(q ≠ 1) . Lúc này (v i b, e như trên) do vn +1 = qvn 1− q 2 (q − 1) nên ( vn ) là c p s nhân có công b i q. Suy ra vn = v1 .q n−1 = (u1 + aq + dq − d ).q n−1 , t ñó ta tính ñư c s h ng (q − 1)2 aq + dq − d n −1 a.n d −a−qd t ng quát c a ( un ) là un = (u1 + ).q + + (n ≥ 2). (q − 1)2 1−q (q −1)2  n(n − 1) c + 2 a + (n − 1)d ,  khi q = 1 V y, s h ng t ng quát c a ( un ) ñã cho là : un =  .  (c + aq + dq − d n−1 a.n d −a−qd ).q + + , khi q ≠ 1   (q − 1)2 1−q (q −1)2 BÀI T P 3 + 2u n u −1 3) Cho (u n ) : u1 = 0, u n +1 = , ∀n ∈ ℕ *. ð t v n = n . Ch ng minh (v n ) là c p s nhân và tìm u n . 4 + un un + 3 1 (n + 1)u n u 4) Cho (u n ) : u1 = , u n +1 = , ∀n ∈ ℕ *. ð t v n = n . Ch ng minh (v n ) là c p s nhân và tìm u n . 3 3n n 5) Cho (u n ) : u1 = 1, u n +1 = u n + 2.(1 + 3 ), ∀n ∈ ℕ *. ð t v n = u n − 3n. Ch ng minh dãy s (v n ) là c p s n c ng và tìm u n . 6) Nêu cách xác ñ nh s h ng t ng quát c a dãy s ( u n) cho b i u1 = c, un+1 = q.un + P (n), ∀n ∈ ℕ * , ñó c, q là các h ng s , còn P(n) là m t ña th c b c k cho trư c. 3.5. TUY N TÍNH HOÁ M T S DÃY PHI TUY N px + q a) V i dãy s (x n ) cho b i x1 = a, x n +1 = n , ∀n ∈ ℕ*, và a, p, q, r, s là các h ng s , ta xét hai dãy (u n ), rx n + s u (v n ) tho mãn u1 = a, v1 = 1, u n +1 = pu n + qv n , v n +1 = ru n + sv n , ∀n ∈ ℕ*, thì x n = n . T ñó tìm ra u n , v n , x n . vn x2 + d b) V i dãy s (x n ) cho b i x1 = a, x n +1 = n , ∀n ∈ ℕ*, và a, d là các h ng s , d ≠ 0, ta xét hai dãy (u n ), 2x n u (v n ) tho mãn u1 = a, v1 = 1, u n +1 = u n + dv 2 , v n +1 = 2u n v n , ∀n ∈ ℕ*, thì x n = n . T ñó tìm ra u n , v n , x n . 2 n vn xn 1 VD7. Cho dãy (x n ) có x1 = 1, x n +1 = , ∀n ∈ ℕ*, tìm x n và ch ng minh x n < , ∀n ≥ 2. 2 + xn n u HD. Xét hai dãy (u n ), (vn ) tho mãn u1 = v1 = 1, u n +1 = u n , v n +1 = u n + 2v n , ∀n ∈ ℕ*, thì x n = n . Ta th y vn x n = 1, ∀n ∈ ℕ *. Và vn+1 = 2vn +1, ∀n ∈ ℕ* ⇔ vn+1 +1 = 2(vn +1), ∀n ∈ ℕ*. Suy ra vn +1 = 2.(vn −1 +1) = 22 (vn −2 +1) = xa.nguyenvan@gmail.com
  • 5. www.VNMATH.com TÂM SÁNG – CHÍ B N u 1 = ... = 2n −1 (v1 + 1) = 2n ⇒ v n = 2n − 1, ∀n ∈ ℕ *. V y x n = n = , ∀n ∈ ℕ *. Ta có 2n = C0 + C1 + ...Cn > n n n n vn 2 − 1 1 1 > C0 + C1 = 1 + n, ∀n ≥ 2, t c là x n = n n < , ∀n ≥ 2. n 2 −1 n 2x n + 1 VD8. Cho dãy (x n ) có x 0 = 2, x n +1 = , ∀n ∈ ℕ, tìm x n và tìm ph n nguyên c a S = x1 + x 2 + ... + x n . 2 + xn u HD. Xét hai dãy (u n ), (v n ) tho mãn u 0 = 2, v0 = 1, u n +1 = 2u n + v n , v n +1 = u n + 2v n , ∀n ∈ ℕ, thì x n = n . Ta vn có u1 = 2u 0 + v0 = 5, u n + 2 = 2u n +1 + vn +1 = 2u n +1 + u n + 2vn = 2u n +1 + u n + 2(u n +1 − 2u n ) ⇒ u n + 2 = 4u n +1 − 3u n , 1 3 phương trình x 2 − 4x + 3 = 0 ⇔ x = 1, x = 3, nên u n = A + B.3n , ∀n ∈ ℕ. Do u 0 = 2, u1 = 5 nên A = , B = . 2 2 3n +1 + 1 3n +1 − 1 3n +1 + 1 Suy ra un = , ∀n ∈ ℕ. Tính ñư c vn = , ∀n ∈ ℕ. V y xn = , ∀n ∈ ℕ. ð t 2 2 3n +1 − 1 32 + 1 33 + 1 3n + 1 3n +1 + 1 3n +1 + 1 2 S = x1 + x 2 + ... + x n = + + ... + + . Lưu ý x n = = 1+ > 1, ∀n ∈ ℕ. Ta có 2 3 n n −1 n +1 n +1 3 −1 3 −1 3 −1 3 −1 3 −1 3 −1 1 (1 + 2)n +1 = C0 +1 + C1 +1.2 + Cn +1.22 + ... + Cn +1.2n +1 ≥ C0 +1 + C n +1.22 = 1 + (n + 1)n. .4 = 2n(n + 1) + 1, ∀n ∈ ℕ *. n n 2 n +1 n 2 2 2 1 1 1 D n t i 3n +1 − 1 ≥ 2n(n + 1), ∀n ∈ ℕ* ⇒ x n = 1 + ≤ 1+ = 1+ − , ∀n ∈ ℕ *. Do v y v i 3n +1 − 1 n(n + 1) n n +1 1 1 1 1 1 1 1 ∀n ∈ ℕ * thì S = x1 + x 2 + ... + x n ≤ (1 + − ) + (1 + − ) + ... + (1 + − ) = n +1− < n + 1. M t khác 1 2 2 3 n n +1 n +1 S = x1 + x 2 + ... + x n > n, ∀n ∈ ℕ *. Như v y n < S < n + 1, ∀ ∈ ℕ*, nên [S] = n, ∀∈ ℕ *. 4. M T S BÀI TOÁN LIÊN QUAN T I TÍNH CH T C A DÃY S VD9. Cho các s dương a1, a 2 ,..., a13 tho mãn a1 + a 2 + ... + a13 ≥ 13. Ch ng minh dãy (un) cho b i u n = a1 + a n + ... + a13 , ∀n ∈ ℕ*, là dãy tăng không nghiêm ng t. n 2 n HD. V i m i s dương a và s nguyên dương n ta có (a n − 1)(a − 1) ≥ 0 nên a n +1 − a n ≥ a − 1. T ñó suy ra u n +1 − u n = (a1 +1 + a n +1 + ... + a13 1) − (a1 + a n + ... + a13 ) ≥ a1 + a 2 + ... + a13 − 13 ≥ 0, ∀n ∈ℕ*, hay un+1 ≥ un , ∀n ∈ ℕ *. n 2 n+ n 2 n V y (un) là dãy s tăng không nghiêm ng t. −1 VD10. Ch ng minh dãy s (un) cho b i u1 = 1, u n = , ∀n = 2,3, 4... là dãy s gi m và b ch n. 3 + u n −1 −3 + 5 −3 + 5 HD. * Trư c h t ta ch ng minh u n > , ∀n ∈ ℕ *. Th t v y, v i n = 1 thì u1 = 1 > . Gi s 2 2 −3 + 5 3+ 5 1 2 3− 5 −1 −3 + 5 uk > . Khi ñó 3 + u k > ⇒ < = ⇒ u k +1 = > . Theo nguyên lí 2 2 3 + uk 3 + 5 2 3 + uk 2 −3 + 5 −3 + 5 qui n p toán h c, ta có u n > , ∀n ∈ ℕ*, t c là (un) b ch n dư i b i . 2 2 1 u 2 + 3u n + 1 −3 + 5 * Bây gi ta xét hi u u n − u n +1 = u n + = n > 0, ∀n ∈ ℕ*, do u n > , ∀n ∈ ℕ *. V y (un) 3 + un 3 + un 2 là dãy s gi m. * Vì (un) gi m nên 1 = u1 > u 2 > ... > u n > u n +1 > ... suy ra (un) b ch n trên b i 1. V y (un) là dãy s b ch n. xa.nguyenvan@gmail.com
  • 6. www.VNMATH.com TÂM SÁNG – CHÍ B N −a Nh n xét. Ta có k t lu n tương t ñ i v i dãy s (un) cho b i u1 = α , u n = , ∀n = 2,3, 4... , v i a, b, c b + cu n −1 −b + b 2 − 4ac dương, b 2 − 4ac > 0, α > . 2c 1 a VD11. Xét tính ñơn ñi u và b ch n c a dãy s (un) có x1 > 0, x n +1 = (x n + ), ∀n ∈ ℕ*, ñó a > 0 là h ng s . 2 xn 1 a HD. Do x1 > 0, a > 0 và x n +1 = (x n + ), ∀n ∈ ℕ*, nên b ng qui n p ta ch ng minh ñư c x n > 0, ∀n ∈ ℕ*, 2 xn 1 a x t c là dãy (un) b ch n dư i. Áp d ng b t ñ ng th c Côsi ta có x n = (x n −1 + ) ≥ a , ∀n ≥ 2. Do ñó n +1 = 2 x n −1 xn 1 a 1 a = + ≤ + = 1, ∀n ≥ 2. Suy ra (un) là dãy gi m không nghi m ng t, k t s h ng th 2 tr ñi. D th y 2 2x 2 2 2a n (un) b ch n trên. V y (un) là dãy b ch n. BÀI T P. 7) Cho dãy (x n ) : x1 = 7, x 2 = 50, x n + 2 = 4x n +1 + 5x n − 1975, ∀n ∈ ℕ *. Ch ng minh x1996 ⋮1997. 8) Cho dãy các s nguyên (x n ) : x1 = 15, x 2 = 35, x 3 = 405, x n + 3 = 6x n + 2 + 13x n +1 − 42x n , ∀n ∈ ℕ *. Tìm nh ng s h ng c a dãy mà ch s t n cùng c a s h ng ñó là s 0. 5. GI I H N C A DÃY S Chúng tôi lưu ý kí hi u n là m t bi n s nguyên dương, còn n0 là m t h ng s nguyên dương (tr trư ng h p có chú thích c th ). M t dãy s có gi i h n h u h n ñư c g i là dãy s h i t , n u nó có gi i h n vô c c ho c không có gi i h n thì ta nói nó phân kì. Khi xét gi i h n c a dãy s (un) ta có th ch xét các s h ng c a dãy k t s h ng th n0 tr ñi, t c là vi c thay ñ i h u h n s h ng ñ u tiên c a dãy s không làm nh hư ng ñ n tính h i t , và không làm nh hư ng ñ n gi i h n (n u có) c a dãy s ñó. Gi i h n c a dãy s (n u có) là duy nh t. T c là n u limun = u thì limun+k = limun-k = u, v i k là s nguyên dương tùy ý. limun = 0 ⇔ lim|un| = 0. limun = u ⇔ lim|un – u| = 0. limun = u ⇔ limu2n = limu2n+1 = u . 1 1 N u limun = u thì lim|un| = |u| và lim u k = uk (k nguyên dương), lim n = (u ≠ 0) . un u N u un < un+1 < M (∀n ≥ n0) thì (un) h i t , limun = u ≤ M, và un < u (∀n ≥ n0). N u un ≤ un+1 ≤ M (∀n ≥ n0) thì (un) h i t , limun = u ≤ M, và un ≤ u (∀n ≥ n0). N u un ≤ un+1(∀n ≥ n0) và (un) không b ch n trên thì limun = + ∞. N u un > un+1 > m (∀n ≥ n0) thì (un) h i t , limun = u ≥ m, và un > u (∀n ≥ n0). N u un ≥ un+1 ≥ m (∀n ≥ n0) thì (un) h i t , limun = u ≥ m, và un ≥ u (∀n ≥ n0). N u un ≥ un+1(∀n ≥ n0) và (un) không b ch n dư i thì limun = – ∞. M t dãy s h i t thì b ch n. N u xn ≤ yn ≤ zn (ho c xn < yn < zn ) v i ∀n ≥ n0, ñ ng th i limxn = limzn = a thì limyn = a (nguyên lí gi i h n k p). N u un ≥ 0 (ho c un > 0) v i ∀n ≥ n0, và limun = u thì u ≥ 0 và lim u n = u . xa.nguyenvan@gmail.com
  • 7. www.VNMATH.com TÂM SÁNG – CHÍ B N Gi s un ≤ vn (ho c un < vn) v i ∀n ≥ n0. Khi ñó: • N u limun = u, limvn = v thì u ≤ v. • N u limun = + ∞ thì limvn = + ∞. • N u limvn = – ∞ thì limun = – ∞. n un  1  1  Ta có lim 1 +  = e. N u limun = + ∞ ho c limun = – ∞ thì lim 1 +  = e.  n  un  N u (un) b ch n và limvn = 0 thì lim(unvn) = 0. 2a n −1.a n +1 VD12. Cho dãy s (an) th a mãn a n = , ∀ n > 1. Tìm lim an. a n −1 + a n +1 2a k .a k + 2 2a .a HD. N u t n t i s nguyên dương k sao cho ak = 0 thì a k+1 = = 0 , suy ra a k+2 = k+1 k +3 = 0 , và a k + a k +2 a k+1 + a k +3 2a k .a k + 2 1 d n t i a k+1 = không có nghĩa. Do v y an ≠ 0, ∀ n ∈ N *. Bây gi ta ñ t un = (∀ n ∈ N *), thì un a k + a k +2 an 1 ≠ 0 (∀ n ∈ N *), thay vào ñ ng th c ñ bài ta ñư c un = (un-1+ un+1), ∀n >1, suy ra (un) là c p s c ng có công 2 a1 sai d, và s h ng t ng quát un = u1 + (n – 1)d ≠ 0, v i m i n ∈ N *. Như v y a n = (∀ n ∈ N *). 1 + (n − 1)da1 N u d = 0 (⇔ u1 = u2 = …⇔ a1 = a2 = …) thì an = a1(∀ n ∈ N *) và lim an = a1. N u d ≠ 0 (⇔ các s h ng c a a1 dãy (un) phân bi t ⇔ các s h ng c a dãy (an) phân bi t) thì lim a n = lim = 0. 1 + ( n − 1) d a 1 V y, n u các s h ng c a (an) b ng nhau thì lim an = a1, n u các s h ng c a dãy (an) phân bi t thì lim an = 0.  0, khi q < 1   1, khi q = 1 n VD13. Ch ng minh r ng limq =  + ∞, khi q > 1 .  không tô`n tai, khi q ≤ −1 .  HD. N u q = 1 thì có ngay limqn = 1. Và n u q = 0 thì cũng có ngay limqn = 0. 1 1 N u 0 <|q| < 1 thì = 1 + a (a > 0)⇒ n = (1 + a)n = C0 + a.C1 + ... + a n .Cn ≥1 + na > 0, v i ∀ n ∈ N *. n n n q q n 1 1 Do ñó 0 < q ≤ , v i ∀ n ∈ N *. Vì a > 0 nên lim = 0. Theo nguyên lí gi i h n k p thì lim|q|n = 0, 1 + na 1 + na hay limqn = 0. 1 1 N u q > 1 thì 0 < < 1 nên theo ch ng minh trên lim n = 0. Ta ñi ñ n limqn = + ∞. q q N u q = –1 thì limq2n = 1 còn limq2n+1 = –1 nên không t n t i limqn. N u q < – 1 thì q2 > 1 nên limq2n = lim(q2)n = + ∞, và limq2n+1 = lim[q.(q2)n] = – ∞, vì th không t n t i limqn. VD14. Cho dãy s (an) th a mãn an ≤ an+1 – a 2 +1 , ∀n ∈ ℕ * . Tìm gi i h n liman. n xa.nguyenvan@gmail.com
  • 8. www.VNMATH.com TÂM SÁNG – CHÍ B N 1 , ∀n ∈ℕ * . T c là (an) là dãy b ch n trên và không HD. T an ≤ an+1 – a 2 +1 , ∀n ∈ ℕ * , suy ra an ≤ an+1 và an ≤ n 4 gi m. Do ñó (an) có gi i h n h u h n liman = a. L y gi i h n h n hai v b t ñ ng th c ñ bài cho, ñư c a ≤ a – a2, hay a = 0. V y liman = 0. VD15. Cho dãy s (xn) th a mãn x1 = 3, x n+1 − 3x n +1 = 2 + x n , ∀ n ∈ N *. Tìm gi i h n lim xn. 3 HD. Ta th y x1 = 3 > 2. Gi s xk > 2, lúc này x 3 − 3x k +1 = 2 + x k > k+1 2 + 2 = 2 nên x 3 − 3x k +1 − 2 > 0 ⇔ (xk+1 + 1)2(xk+1 – 2) > 0 ⇔ xk+1 > 2. T c là xn > 2, ∀ n ∈ N *. Xét hàm s f(t) = t3 – 3t, k+1 có f ’(t) = 3t2 – 3 = 3(t + 1)(t – 1) > 0, ∀ t > 2, suy ra f(t) ñ ng bi n trên kho ng (2; + ∞). Ki m tra th y x1 − 3x1 = 18 3 > x 3 − 3x 2 = 5 ⇒ 2 f(x1) > f(x2) ⇒ x1 > x2. Gi s xk > xk+1 ⇒ 2 + x k > > 2 + x k+1 ⇒ x k+1 − 3x k +1 > x 3 − 3x k + 2 ⇒ f(xk+1) > f(x k +2) ⇒ xk+1 > xk+2. Do ñó xn > xn+1 v i ∀ n ∈ N *. Dãy 3 k+2 (xn) gi m và b ch n dư i b i 2 nên t n t i gi i h n h u h n lim xn = x ≥ 2. L y gi i h n hai v ñ ng th c ñ bài ta ñư c x3 – 3x = 2 + x ⇔ x6 – 6x4 + 9x2 = x + 2 (vì x ≥ 2 nên x3 – 3x > 0) ⇔ x6 – 6x4 + 9x2 – x –2=0⇔ 5 4 3 2 2 3 3 (x – 2).(x + 2x – 2x – 4x + x + 1) = 0 ⇔ (x – 2).(x (x – 4) + 2x (x – 1) + x + 1) = 0 ⇔ x = 2 (do x ≥ 2). V y limxn = 2. VD16. Tính gi i h n: [(2+ 3) n ] a) lim , trong ñó [x] là ph n nguyên c a s th c x, t c là s nguyên l n nh t không vư t quá x. (2+ 3) n c 1 3 5 2n − 1 n 1 b) lim( + 2 + 3 + ... + n ). c) lim n (a > 1, c > 0). d)lim n n . e) lim n . 2 2 2 2 a n! (2 + 3) + (2 − 3) n n HD. a) ð t a n = , nh công th c khai tri n nh th c Niuton ta th y ngay an là s nguyên 2 dương. M t khác – 1< – (2 − 3) < 0 nên có [(2+ 3) n ] = [2an – (2 − 3) ] = 2an +[– (2 − 3)n ] = 2an – 1. n n [(2+ 3) n ] 2a n − 1 (2 + 3)n + (2 − 3) n − 1 1 1 V y lim = lim = lim = lim(1 + − ) = 1. (2+ 3) n (2+ 3) n (2+ 3)n (7 + 4 3) n (2+ 3) n Nh n xét. V i x, y, n, r nguyên dương, r không là s chính phương thì t n t i hai dãy s nguyên dương (an) và (bn) sao cho (x +y r )n = an + bn r ; (x - y r )n = an - bn r ; v i (x + y r)n + (x − y r)n (x + y r)n − (x − y r)n an = , bn = . 2 2 1 3 5 2n − 1 3 5 2n − 1 b) Ta ñ t xn = + 2 + 3 + ... + n ⇒ 2xn = 1 + + 2 + ... + n-1 . Ta có xn = 2xn – xn = 2 2 2 2 2 2 2 1 1 1 1 − 2n 1 1 n 1 1 =2+ + 2 + ... + n −2 + n = 3 – n − 2 + n – n −1 . D th y lim n − 2 = lim n = 0. V i m i n > 2 2 2 2 2 2 2 2 2 2 n−1 n 2 − 3n + 2 ta có 2n −1 = Cn−1 + Cn−1 + Cn−1 + ... + Cn−1 ≥ C 2 −1 = > 0. T ñó suy ra 0 < n < 2n 0 1 2 n , ∀ n > 2. n −1 2 2 n − 3n + 2 2 2n n Mà lim = 0 nên lim n −1 = 0. V y lim xn = 3. n 2 − 3n + 2 2 xa.nguyenvan@gmail.com
  • 9. www.VNMATH.com TÂM SÁNG – CHÍ B N c  c c 1 n  n =  n  ñó b = a c – 1 > 0. Nh n th y v i m i n ≥ 2 thì c) V i hai s a > 1 và c > 0 ta bi n ñ i n = n a    (1+b) n   c    a  (n 2 − n)b 2 n 2 (1 + b) = Cn + bCn + b Cn + ... + b Cn ≥ b Cn = n 0 1 2 2 n n 2 2 > 0 suy ra 0 < < , ∀ n ∈ N *. 2 (1+b) (n − 1)b 2 n c 2 n nc  n  T ñây và do lim = 0 nên lim = 0, d n t i lim n = lim   = 0. (n − 1)b 2 (1+b)n a  (1+b)n  n  1  C1 C2 C3 Cn n − 1 n 2 − 3n + 2 d) V i m i n ≥ 9 ta có 1+  = C0 + n + n + n + ... + n ≥ 1 + n + n + =  n n n n n nn 2 6 n 1 1 n n n 1 n n n n n n n( n − 3) 1 = + n+ + + > + =n+ − =n+ ≥ n . D n ñ n 1 < n n < 1+ v i 2 2 2 6 3 n 2 6 6 2 6 n 1 m i n ≥ 9. Mà lim (1 + ) = 1 nên suy ra lim n n = 1. n log a n Nh n xét. V i a >0, a ≠ 1, thì lim = 0. n 1 1 e) Trư c h t ta có n! = 1.2.3…n ≤ nn ⇒ ≤ , ∀ n ∈ N *. M t khác ∀k = 1, n ta luôn có n n n! (n − k)(k − 1) ≥ 0 ⇒ k(n − k + 1) ≥ n , cho k ch y t 1 ñ n n ta thu ñư c n b t ñ ng th c mà hai v ñ u dương, nhân 1 1 n b t ñ ng th c này, v v i v tương ng, ta ñư c (n!)2 ≥ nn hay n ≤ , ∀n ∈ ℕ* . T n! n 1 1 1 1 1 1 ≤n ≤ , ∀n ∈ ℕ* , và lim = lim = 0 suy ra lim n = 0. n n! n n n n! un VD17. Cho dãy s (un) th a mãn u1 = – 2, un+1 = , ∀ n ∈ N *. 1 − un a. Ch ng minh un < 0, ∀ n ∈ N *. 1+ u n b. ð t vn = , ∀ n ∈ N *. Ch ng minh (vn) là c p s c ng. un c. Tìm công th c s h ng t ng quát c a (un), (vn), và tính các gi i h n limun, limvn. HD. a) Ta ch ng minh un < 0 (∀ n ∈ N *) b ng phương pháp quy n p toán h c. Rõ ràng u1 = – 2 < 0. Bây gi gi uk s uk < 0 ⇒ 1 – uk > 0. D n t i uk+1 = < 0. V y un < 0 (∀ n ∈ N *). 1 − uk 1+ u n 1 1− 2 1 un b) ð t vn = thì vn ≠ 1 và un = . Ta có v1 = = . T un+1 = ta có un vn − 1 −2 2 1− un 1 1 1 1 = : (1 − ) hay vn+1 = vn – 1 (∀ n ∈ N *). V y (vn) là c p s c ng có s h ng ñ u tiên v1= , v n+1 − 1 v n − 1 vn − 1 2 công sai d = –1. xa.nguyenvan@gmail.com
  • 10. www.VNMATH.com TÂM SÁNG – CHÍ B N 3 1 1 2 c) T câu b ta có ngay vn = – n, và un = = hay un = (∀ n ∈ N *). Như v y limun = 0, 2 vn − 1 3 − n − 1 1 − 2n 2 limvn = – ∞. 1 VD18. Cho dãy s (un) th a mãn 0 < un < 1 và un+1 < 1 – 1 v i m i n ∈ N *. Ch ng minh un > , ∀ n ∈ N *. 4u n 2 1 1 HD. T 0 < un < 1 và un+1 < 1 – suy ra un(1 – un+1) > (∀ n ∈ N *). Áp d ng b t ñ ng th c Côsi cho hai s 4u n 4 un và 1 – un+1 ta có un + (1 – un+1) ≥ 2 u n (1 − u n+1 ) > 2. 1 = 1 hay u n + 1 − u n+1 > 1 hay u n > u n+1 (∀ n ∈ N *). 4 Dãy (un) gi m và b ch n nên có gi i h n h u h n lim u n = u ∈ [0; 1] . L y gi i h n hai v c a b t ñ ng th c 1 1 1 1 un(1 – un+1) > ta ñư c u(1 – u) > ⇔ u = . T c là limun = . 4 4 2 2 1 Bây gi ta ñi ch ng minh un > (∀ n ∈ N *) b ng phương pháp ph n ch ng. Gi s t n t i s nguyên dương k 2 1 1 1 sao cho uk ≤ 1 . Lúc này ≥ uk > uk+1 > … > un+k > … , ∀ n ∈ N *. Suy ra ≥ uk > uk+1≥ lim un + k = . 2 2 2 n→+∞ 2 1 ði u này vô lí. V y un > (∀ n ∈ N *). 2 BÀI T P 9) Tính gi i h n: 13 33 53 (2n − 1)3 13 + 23 + ... + n 3 1) lim( 3 + 3 + 3 + ... + ). 2) lim . n n n n3 n4 1 1 1 1 3 2n − 1 3) lim( + + ... + ). 4) lim( . ... ). 1.2 2.3 n(n + 1) 2 4 2n 1 1 1 1 1 5) lim(cos + a.sin )n. 6) lim( + + ... + ). n n n2 +1 n2 + n n2 + n 11 + 22 + ... + n n an 7) lim . 8) lim (v i a > 0). nn n! 1 1 1 9) lim 1 + 2n + ... + 2010n . n n 10) lim( 1 + + + ... + ). 2 3 n 10) Dãy s (xn) có x1 = a >0, x 2 − x n = a (∀ n ∈ N *). Tìm limxn. n+1 11) Dãy s (an) có 0 < an < 1 và an+1 = an(2 – an) v i m i n ∈ N *. Tìm limxn. 6. DÃY S SINH B I PHƯƠNG TRÌNH VD19. Cho s nguyên dương n. Ch ng minh r ng phương trình xn+1 = x + 1 có m t nghi m dương duy nh t, kí hi u là xn. Tìm limxn. HD. Trư c h t n u x > 0 thì x + 1 > 1, t phương trình xn+1 = x + 1 ⇒ xn+1> 1 ⇒ x > 1. Do ñó n u phương trình xn+1 = x + 1 có nghi m dương thì nghi m ñó s l n hơn 1. Ta xét hàm s fn(x) = xn+1 – x – 1 v i x > 1, có ñ o hàm fn’(x) = (n + 1)xn –1 > (n + 1) – 1 = n > 0 v i m i x > 1. Suy ra fn(x) ñ ng bi n trên kho ng (1; + ∞). T ñây, và tính liên t c c a fn(x), và fn(1) = – 1< 0, lim f n (x) = +∞ , ta k t lu n phương trình xn+1 = x + 1 có nghi m x→+∞ dương duy nh t xn. T t nhiên xn > 1. Do fn+1(x) liên t c, fn+1(1) = – 1< 0, fn+1(xn) = x n+2 − x n − 1 > x n − x n − 1 = n n+1 xa.nguyenvan@gmail.com
  • 11. www.VNMATH.com TÂM SÁNG – CHÍ B N = fn(xn) = 0, nên phương trình xn+2 = x + 1 có nghi m dương duy nh t xn+1 và 1 < xn+1< xn. Dãy (xn) gi m và b ch n dư i b i 1 nên có gi i h n h u h n limxn = a ≥ 1. Do xn là nghi m dương c a phương trình xn+1 = x + 1 nên 1 1 − 1 = 0 ⇒ x n = (1 + x n ) n +1 ⇒ limx n = lim(1 + x n ) n +1 = (1 + a ) = 1. 0 x n+1 − x n n VD20. Cho n là m t s nguyên dương > 1. Ch ng minh r ng phương trình xn = x + 1 có m t nghi m dương duy nh t, ký hi u là xn. Ch ng minh r ng xn d n v 1 khi n d n ñ n vô cùng và tìm lim n( x n − 1) . n→ ∞ HD. Rõ ràng xn > 1. ð t fn(x) = xn – x – 1. Khi ñó fn+1(1) = - 1 < 0 và fn+1(xn) = xnn+1 – xn – 1 > xnn – xn – 1= =fn(xn) = 0. T ñó ta suy ra 1 < xn+1 < xn . Suy ra dãy {xn} có gi i h n h u h n a. Ta ch ng minh a = 1. Th t v y, gi s a > 1. Khi ñó xn ≥ a v i m i n và ta tìm ñư c n ñ l n sao cho: xnn ≥ an > 3 và xn + 1 < 3, mâu thu n vì fn(xn) = 0. ð gi i ph n cu i c a bài toán, ta ñ t xn = 1 + yn v i lim yn = 0. Thay vào phương trình fn(xn) = 0, ta ñư c (1+yn)n = 2 + yn. L y logarith hai v , ta ñư c nln(1+yn) = ln(2+yn). T ñó suy ra lim nln(1+yn) = ln2. Nhưng lim ln(1+yn)/yn = 1 nên t ñây ta suy ra lim nyn = ln2, t c là lim n( xn − 1) = ln 2. n →∞ 1 1 1 VD21. Ký hi u xn là nghi m c a phương trình: + + ... + = 0 thu c kho ng (0, 1) x x −1 x−n a) Ch ng minh dãy {xn} h i t b) Hãy tìm gi i h n ñó. HD. Rõ ràng xn ñư c xác ñ nh 1 cách duy nh t, 0 < xn < 1. Ta có fn+1(xn) = fn(xn) + 1/(xn-n-1) = 1/(xn-n-1) < 0, trong khi ñó fn+1(0+) > 0. Theo tính ch t c a hàm liên t c, trên kho ng (0, xn) có ít nh t 1 nghi m c a fn+1(x). Nghi m ñó chính là xn+1. Như th ta ñã ch ng minh ñư c xn+1 < xn. T c là dãy s {xn} gi m. Do dãy này b ch n dư i b i 0 nên dãy s có gi i h n. Ta s ch ng minh gi i h n nói trên b ng 0. ð ch ng minh ñi u này, ta c n ñ n k t qu quen thu c sau: 1 + 1/2 + 1/3 + … + 1/n > ln(n) (Có th ch ng minh d dàng b ng cách s d ng ñánh giá ln(1+1/n) < 1/n). Th t v y, gi s lim xn = a > 0. Khi ñó, do dãy s gi m nên ta có xn ≥ a v i m i n. Do 1 + 1/2 + 1/3 + … + 1/n +∞ khi n + ∞ nên t n t i N sao cho v i m i n ≥ N ta có 1 + 1/2 + 1/3 + … + 1/n > 1/a. Khi ñó v i n ≥ N ta có 0 = 1 + 1 + ... + 1 < 1 + 1 + 1 + ... + 1 < 1 − 1 = 0 . Mâu thu n. V y ta xn xn − 1 xn − n xn −1 −2 −n a a ph i có lim xn = 0. VD22. (VMO 2007) Cho s th c a > 2 và fn(x) = a10xn+10 + xn + …+x + 1. a) Ch ng minh v i m i s nguyên dương n, phương trình fn(x) = a luôn có ñúng m t nghi m dương duy nh t. b) G i nghi m ñó là xn, ch ng minh r ng dãy {xn} có gi i h n h u h n khi n d n ñ n vô cùng. HD. K t qu c a câu a) là hi n nhiên vì hàm fn(x) tăng trên (0, +∞). D dàng nh n th y 0 < xn < 1. Ta s ch ng minh dãy xn tăng, t c là xn+1 > xn. Tương t như nh ng l i gi i trên, ta xét fn+1(xn) = a10xnn+11 + xnn+1 + xnn + … + x + 1 = xnfn(xn) + 1 = axn + 1. 10 Vì ta ñã có fn+1(1) = a + n + 1 > a nên ta ch c n ch ng minh axn + 1 < a là s suy ra xn < xn+1 < 1. Như v y, c n ch ng minh xn < (a-1)/a. Th t v y, n u xn ≥ (a-1)/a thì n +1  a −1  n +10 1−   n n  a −1   a −1   a −1  +  a  fn (xn ) ≥ a10   = (a −1)10   + a − (a −1)   >a  a  a −1  a   a  1− a (do a – 1 > 1). V y dãy s tăng {xn} tăng và b ch n b i 1 nên h i t . 1 1 1 1 VD23. (VMO 2002) Cho n là m t s nguyên dương. Ch ng minh r ng phương trình + + ... + 2 = x − 1 4x − 1 n x −1 2 có m t nghi m duy nh t xn > 1. Ch ng minh r ng khi n d n ñ n vô cùng, xn d n ñ n 4. HD. ð t fn(x) như trên và g i xn là nghi m > 1 duy nh t c a phương trình fn(x) = 0. Ta có 1 1 1 1 1 1 1 1 1 1 1 1 1 1 1 1 1 fn (4) = + + ... + 2 − = + + ... + − =  − + − + ... + − − =− 4 −1 16 −1 4n −1 2 1.3 3.5 (2n −1)(2n +1) 2 2  1 3 3 5 2n −1 2n  2 4n xa.nguyenvan@gmail.com
  • 12. www.VNMATH.com TÂM SÁNG – CHÍ B N Áp d ng ñ nh lý Lagrange, ta có : 1/4n = |fn(xn) – f(4)| = |f’(c)||xn-4| v i c thu c kho ng (xn, 4). Nhưng do 1 4 1 | f n '(c) |= + + ... > n ên t ñây |xn – 4| < 9/4n, suy ra lim xn = 4. (c − 1) 2 (4c − 1) 2 9 VD24. Cho n là m t s nguyên dương > 1. Ch ng minh r ng phương trình xn = x2 + x + 1 có m t nghi m dương duy nh t, ký hi u là xn. Hãy tìm s th c a sao cho gi i h n lim n ( x n − x n +1 ) t n t i, h u h n và khác 0. a n→∞ HD. ð t Pn(x) = xn – x2 – x – 1. Ta có Pn+1(x) = xn+1 – x2 – x – 1 = xn+1 – xn + Pn(x) = xn(x-1) + Pn(x). T ñó Pn+1(xn) = xnn(xn-1) + Pn(xn) = (xn2+xn+1)(xn-1) = xn3 – 1. Áp d ng ñ nh lý Lagrange, ta có: (xn2+xn+1)(xn – 1) = Pn+1(xn) – Pn+1(xn+1) = (xn – xn+1)Pn+1’(c) v i c thu c (xn+1, xn), Pn+1’(x) = (n+1)xn – 2x – 1. T ñó (n+1)(xn+1+1+1/xn+1) – 2xn+1 – 1 = Pn+1’(xn+1) < Pn+1’(c) < Pn+1’(xn)= (n+1)(xn2+xn+1) – 2xn – 1. T ñây, v i lưu ý lim xn = 1, ta suy ra : lim Pn +1 '(c ) = 3 .Ti p t c s d ng lim n(xn – 1) = 3, ta suy ra: n →∞ n lim nPn +1 (c)( xn − xn +1 ) = lim n( xn + xn + 1)( xn − 1) = 3ln(3) ' 2 n →∞ n →∞ Pn' +1 (c) P ' (c ) ⇔ lim n 2 ( xn − xn +1 ). = 3ln(3) ⇔ lim n 2 ( xn − xn +1 ) lim n +1 = 3ln(3) n →∞ n n →∞ n →∞ n ⇔ lim n 2 ( xn − xn +1 )3 = 3ln(3) ⇔ lim n 2 ( xn − xn +1 ) = ln(3) n →∞ n →∞ V y v i c = 2 thì gi i h n ñã cho t n t i, h u h n và khác 0. D th y v i c > 2 thì gi i h n ñã cho b ng vô cùng và n i c < 2 thì gi i h n ñã cho b ng 0. V y c = 2 là ñáp s duy nh t c a bài toán. BÀI T P 12) V i m i s nguyên dương n phương trình x = n x + 1 có m t nghi m dương duy nh t, kí hi u là xn. Tìm lim(n(xn – 1)). xa.nguyenvan@gmail.com